Location via proxy:   [ UP ]  
[Report a bug]   [Manage cookies]                

Manual Content

Download as pdf or txt
Download as pdf or txt
You are on page 1of 125

CHAPTER I

SIMPLE INTEREST
LEARNING OBJECTIVES

• Gain an understanding on the concept of simple interest.


• Understand how simple interest is applied to business practice.
• Acquire basic knowledge on how to compute for simple interest.
• Understand the concept of time and interest as applied to investments.
• Understand the concept of bank discount and how it is applied to business
practice.
• Gain an understanding on how promissory notes are discounted.
In business practice, an investor who invests capital in a profitable venture
(investment) expects not only an eventual return of the amount he invested but
rather, expects an additional payback. In a simpler sense, a creditor or investor
who lends an amount of money to a debtor expects not only the amount he lent
at maturity or payment date, but also an additional amount to compensate the
return he had given up (opportunity cost) had he invested it into an alternative
profitable undertaking. This additional payment of the debtor (receipt in the point
of view of the creditor-investor) is called interest. Interest therefore, is regarded as
a compensation that a borrower of capital or a debtor pays to a lender for its use. In
simple parlance, it is money’s rent; and since investment involves investor-investee
relationship, interest can be viewed as an expense to the debtor and income on
the part of the creditor. It is to be noted that while this manual is intended for
understanding investments, it is also worthwhile to understand the duality of its
nature, as in this case, the point of view of the debtor is introduced.

Generally, interest is of two types: simple and compound. In this chapter, we will
only be dealing with simple interest and compound interest will be discussed
lengthily in the succeeding chapter.

SIMPLE INTEREST

Simple interest is an interest computed on the amount, commonly known as the


principal, P the borrower received at the time the loan is obtained, and is added to
that amount when the loan becomes due on its term or time, t at a certain interest
rate, r.

Thus, simple interest is computed only once for the entire period of the investment.
However, the recognition of an interest in accounting (whether expense or income,
as the case may be) is applied on the basis of accrual. Hence, an interest (accrued
or incurred) for 3 years is applied evenly for the entire period.

At maturity date, the borrower repays the amount originally borrowed and the
interest. This accumulated amount of the loan is called maturity value. Some
creditors however prefer to collect the interest in advance. The interest deducted in
advance is called bank discount (discussed separately). Simple interest is usually
employed to investments (loans) whose time period is less than a year.

The simple interest on a principal, P is proportional to the time, t, P is invested.


Hence, the amount of interest earned for each period is constant. With regards to
a longer term, e.g. more than a year, the interest per year is multiplied to t to get
the total interest on the investment. From these premises, the formula for interest,
I is derived as:

2
Formula 1: Simple Interest


I = Prt


where:

I interest

P principal/original amount borrowed

r interest rate

t time/term

The triangular diagram below is useful in manipulating Formula 1 for solving


other unknown variables like P, r and t. To use it, cover the unknown variable and
regard vertical line as multiplication and horizontal line as division. For example,
I/Pr will be left if you cover t. This shows that t = I/Pr.

It should be noted that interest rate, unless otherwise stated is assumed to be a rate
applied per annum.

Figure I.1. Simple Interest Formula

Note: Different Mathematics of Investment book authors use different letters to


name the variables in the simple interest formula. In this case, we use I
for simple interest/interest as the case may be (other uses SI, Int), P for
the principal amount, r for the interest rate (which other uses i) and t to
represent time or the term of the investment.

3
It should be noted that basic knowledge in algebra, as in the case of formula
derivation is needed.

Example 1: Yaneth Joyce invests P10,000 for a year. The investment bears interest
rate of 9%?. How much does it earn after its term? How much interest
would the investment yield if its term is 4 years, in 5 years?

Solution: a. We use Formula 1, I = Prt; with t = 1, P = 10,000 and r = 9%

I = (10,000)(0.09)(1)
I = 900

b. with t = 4

I = (10,000)(0.09)(4)
I = 3,600

b. with t = 5

I = (10,000)(0.09)(5)
I = 4,500

Since simple interest applied to a loan is constant, simple interest can be alternatively
computed as:

Interest earned in a year is 900. Interest in 4 years is 4(900) = 3,600 and


interest in 5 years is 5(900) = 4,500.

Example 2: Mark Melvin accumulated an interest amounting to P4,500 on a loan


extended to Mark Joseph at a rate of 12% for 3 years. How much was
borrowed by Mark Joseph from Mark Melvin?

Solution: Given that t = 3, r= 12% and I = 4,500, then from I = Prt

P = I/rt
P = 4,500/[(0.12)(3)]
P = 12,500

Example 3. Melchor, Jr. borrowed P12,000 from Mary Jane to be paid after 2 years.
At what interest rate would make Melchor, Jr. liable for an interest of
P1,800 at the end of the term of the loan made by him?

Solution: Since P = 12,500, I = 1,800 and t =2; using I = Prt:

4
r = [I/Pt] x 100
r = [1,800/(12,000)(2)] x 100
r = 7.5%

Example 4: Lloyd Russell extended a P10,000 amount of loan to Hershey at a


certain interest rate. If the investment had accumulated an interest of
P800 each year for 2 years, what interest rate must have been agreed
by them?

Solution: Since P = 10,000, I = 800 per year, hence I = 1,600 for 2 years term and
t =2; using I = Prt:

r = [I/Pt] x 100
r = [1,600/(10,000)(2)] x 100
r = 8%

Example 5: Elmo extended a P5,000 loan to Bonnie at an interest rate of 8%. If


Elmo received a total interest of 2,000 at the date of maturity, how
long was the agreed term?

Solution: Since P = 5,500, I = 2,000 and r =8%; using I = Prt,

t = I/Pr
t = 2,000/[(5,000)(0.08)]
t = 5 years

THE MATURITY VALUE

Maturity Value, MV or accumulated amount is the amount received (by the creditor,
investor) or paid (by the debtor, investee), as the case may be, after the period of
the investment. The determination of amount of MV when P, r and t are known
is called accumulating P. To accumulate P for t years at the rate r means to find the
amount that would be payable (debtor) or receivable (creditor) at the end of t years
if P is invested at the rate r.

Maturity Value is also called the Future Value, Accumulated Amount or Final Amount.
Hence, as far as simple interest is concerned, the principal, P is the investment’s
present value.

The formula for Maturity Value is as follows:

5
Formula 2: Maturity Value


MV = P + I

From the above formula, we can deduce that:

MV = P + (Prt)

Applying factoring in algebra, we have ;

MV = P(1+rt)


where:

MV maturity value
I interest

P principal/original amount

r interest rate

t time/term

Example 6. Jana borrowed P5,000 from Jerry at a simple interest rate of 9% for
two years. Compute for the interest. After two years, what would be
the loan’s maturity value?

Solution: We use Formula 1 to find the interest, I = Prt; with t = 2, P = 5,000 and
r = 9%

I = (5,000)(0.09)(2)
I = 900

To find the maturity value, we use Formula 2.

MV = 5,000 + 900
MV = 5,900

6
Example 7: Henson borrowed P200,000 from BJ at an interest rate of 8% per
annum to last for 3 years. At the date of maturity, how much would
BJ’s expected cash inflow?

Solution: The problem asks about the value of the investment at maturity date.
It is the expected cash inflow of BJ (outflow from Henson’s point of
view).

With t = 3, P = 200,000 and r = 8%

MV = 200,000[1 + (0.08)(3)]
MV = 248,000

THE CONCEPT OF TIME

The time t in the simple interest formula I = Prt is the time or period between
the time the investment is made and the date of maturity or return date. In our
previous examples, time is exactly expressed as whole year or years. However,
there are instances wherein time t is a fraction of a year, expressed in months or it
could also be expressed in days.

If the time is expressed in months, we express it as a fraction of a year (or years, as


the case may be), assuming a year to contain 12 equal months, i.e. t = 4 months =
4/12 year or 0.33 year. If time is expressed in days, there are two ways on how to
compute for the time: Exact/Actual and Approximate/Estimate time computation.
These two methods are more commonly known as the Exact Interest Method
(Exact/Actual) and the Ordinary Interest Method (Approximate/Estimated).

Exact/Actual time uses the exact number of days in a given month while
Approximate/Estimated time assumes that all months have 30 days, hence a year
contains a total of 360 days.

Example 8: If Mark Melvin borrowed P130,000 from Mark Joseph at 7% interest


for 45 days, how much would be the interest using exact and ordinary
interest methods?

Solution: a. Exact interest method (Actual/Exact)

I = Prt
I = 130,000 x 0.07 x 45/365
I = 1,121.92

b. Ordinary interest method (Approximate/Estimated)

7
I = Prt
I = 130,000 x 0.07 x 64/360
I = 1,617.78

Note that the ordinary interest method yielded a higher interest compared to the
exact interest method.

There are investment problems which provide only the dates when the investment
was made and when it is due. In these cases, the following rules are observed:

1. Actual time is determined by counting every day excluding the loan date
until the maturity date.
2. Approximate time is obtained by assuming that each month has 30 days,
hence in a year, there are 360 days.

Example 9. Find the time(exact and approximate) from January 11, 20XX to
August 23, 20XX assuming that 20XX is not a leap year.

Solution: To find the actual time, the number of days in each of the twelve
months of the year (ignoring leap year) is shown in the table:

The Number of Days in Each Month of the Year

Month Days Month Days Month Days


January 31 May 31 September 30
February 28* June 30 October 31
March 31 July 31 November 30
April 30 August 31 December 31
* In case the date falls on a leap year, February contains 29 days.


Table I.1 Actual number of days in a year

When determining the number of days between two dates, the current practice is
to ignore the beginning date but include the ending date.

In such case, in our example, time is computed therefore as:

31-11 20 days January


28 February
31 March
30 April
31 May
30 June
31 July
23 August
Exact time 224 days

8
Example 10: Find the approximate time from June 30, Y to March 26, 20Y+1.

Solution: Since approximate time proposes that months in a year has an equal
number of days, which is 30 days; hence, a year has 360 days.

To find the approximate time in the example:

30 days July Y
30 August Y
30 September Y
30 October Y
30 November Y
30 December Y
30 January Y+1
30 February Y+1
26 March Y+1
Approximate time 266 days

Alternatively, approximate time can be computed using the following formula:


Formula 3: Approximate time

360(Y2 - Y1) + 30(M2 - M1) + (D2 - D1)

where:
Y2;1 year of second date; first date

M2;1 month of second date; first date

D2;1 day of second date; first date

Assuming that the given dates fall on years 2010 and 2011 respectively.

Thus, 360(2011-2010) + 30(3-6) + (26-30) = 266 days.

Example 11: Count the actual and approximate time from June 15, 2006 to
February 2, 2007. The computation of exact and approximate time in
the given example is compared in the table that follows:

Actual time Approximate time


June, 2006 30-15 15 June, 2006 30-15 15
July 31 July 30
August 31 August 30
September 30 September 30
October 31 October 30
November 30 November 30
December 31 December 30

9
January, 2007 31 January, 2007 30
February 2 February 2
232 days 227 days
Our computation shows that the actual time is longer than the approximate time.
But take note that what we have counted is only the numerator. With regards to
the denominator, we could use, as discussed previously, the exact and ordinary
interest method using 365-days and 360-days respectively.

It may then be deduced that where only the loan date and maturity date are given,
there are four possible time combinations to solve for the interest. Depending on
what the problem asks, any of the following fractions is substituted to variable t in
the simple interest formula I = Prt. These are as follows:

Actual time 232 Actual time 232


1. = 3. =
Exact interest 365 Ordinary interest 360

Approximate time 227 Approximate time 227


2. = 4. =
Exact interest 365 Ordinary interest 360

When the type of interest is not specified in any problem, Banker’s Rule (Actual
Time/Ordinary Interest) is applied and this is commonly used in business practice.
Why?

The reason behind is that this method is more favorable for the lender than
the other methods because the exact number of days is usually larger than the
approximate time (although exceptions do exist) and the divisor is only 360 days.
With this amount of time, the Banker’s Rule yields the highest interest among the
four methods of computing time.

BANK DISCOUNT

In some instances, the creditor would want a guaranty that interest when due are
collected in full, thus, some creditors collect interest in advance for the amount
borrowed by the debtor. This advance interest is called bank discount. A bank
discount is an interest computed on the maturity value of the loan and is deducted
from that amount at loan date to arrive at net proceeds to be received by the
borrower.

Take note that the word discount as used in relation to investment problems is
relatively different with its other uses in commercial transactions. For example,
the word discount frequently refers to a reduction in price to encourage prompt
payment or bulk purchases. In accounting for bonds, bonds payable is said to
have been issued at a discount if its present value at issuance date is less than its
maturity value (investor’s point of view).

10
Bank discount, discount or simple discount is seldom used in transaction extending
over more than a year. Interest is computed as usual but it is then subtracted from
the principal and this amount is loaned to the borrower, hence it is called discount.
Pawnshops usually employ this discounting method.

Illustration: If Wimbledon goes to a bank and apply for a loan of P2,000 for a year
at a rate of 5%, then the bank will give him the amount P2,000 as the
principal. At the end of the year, he will repay the bank the amount
borrowed of P2,000 and interest of P100. Hence, he would have to
pay a total amount of, P2,100.

However, if he borrows the same at a bank discount rate of 5%,


then the bank will collect P100 in advance as interest and the bank
will only give him P2,900 as loan proceeds. At the end of the year,
he will only repay the principal amount of P2,000. No payment for
interest will be made at the maturity date, because it was collected
in advance. However, in essence, the payment is still worth P2,100
inclusive of principal and interest.

In computing for the bank discount, we need to identify the maturity value, time
and the bank discount rate.

The formulas related to bank discount are as follows:

Formula 4: Simple Discount


I = MV x r x t
P = MV-I
P = MV(1-rt)


where:

I bank discount
MV maturity value
P proceeds from the loan

r discount rate

t time

11
Example 12: Darwin applied for a loan of P200,000 at Pinoy National Bank at
12% discount rate for 8 months. Determine the amount of interest
collected in advance. What was Darwin’s proceeds from the loan?
The maturity value is the amount applied for by the borrower. This is the amount
the borrower is expected to pay at maturity date. In this case, MV = P200,000. The
proceeds is the amount of money that is received, net of discount.

The bank discount rate expressed as a percentage is converted to decimal and the
time is expressed as a fraction of a year.

Solution: Using Formula 4, I = MV x r x t, with MV = P200,000, r = 12% and


t = 8 months.

a. To compute for the bank discount

I = MV x r x t
I = 200,000 x 0.12 x 8/12
I = 16,000

b. To compute for the proceeds from the loan

P = MV – I
P = 200,000 – 16,000
P = 184,000

or using the formula P = MV(1-rt)

P = 200,000[1-(0.12 x 8/12)]
P = 184,000

PROMISSORY NOTES

In actual practice, investments are usually supported by instruments to bind


the contract between parties. For example, for every loan extended to a debtor,
it is necessary that a contract be executed supported by an instrument to bind
the indebtedness of the debtor to the lender. Once the debtor agrees and signs in
the agreements stipulated in the contract, he certifies that he has an obligation to
fulfill, and that is to pay the amount borrowed at the agreed payment time. This
contract of indebtedness is usually supported by a promissory note.

A promissory note, as defined in the Negotiable Instruments Law, “is an

12
unconditional promise in writing by one person to another, signed by the maker, engaging
to pay on demand, or at a fixed or determinable future time, a sum certain in money to
order or to bearer”. The definition specifies the elements of a promissory note.
Generally, promissory notes are of two types namely simple interest and bank
discount notes. In order for us to solve problems involving promissory notes, it is
important that we know the elements of a promissory note.

We first illustrate a simple interest promissory note:

SIMPLE INTEREST NOTE

Example 13: On January 03, 2011, Michael Johns borrowed P5,000 from City
Hunter Co.. The loan was approved at 10% simple interest for 3
years. To bind the contract of loan, the following promissory note
was issued by Michael Johns:

The features of the note are the following:

1. The borrower/maker of the note is the party making the promise to pay (Michael
Johns).
2. The payee/lender of the note is the party to whom the promise is made (City
Hunter Co.).
3. The face value/amount borrowed of the note is the sum of money specified
(P5,000).
4. The rate of interest stated as an annual rate based on the face value of the note
(10%).
5. The term of the note is the length of time between the date of issue and when the
note matures (3 years).
6. The due date/date of maturity is the date on which the note is to be paid.
7. The maturity value is the amount payable on the due date of the note.

13
The simple interest on the promissory note could be computed easily by using
the Simple Interest Formula with some modifications; that is, the face value, FV is
substituted to the principal, P in the formula. Thus, I = Prt becomes I = FV x r x t.

I = FV x r x t
I = 5,000 x 0.10 x 3
I = 1,500
The maturity value, MV is computed by adding the interest and the face value of
the note:

MV = FV + I
MV = 5,000 + 1,500
MV = 6,500

On the date of maturity, meaning the payment date, Michael Johns will have to
pay City Hunter Co. the amount of P6,500.

BANK DISCOUNT NOTE

Example 14: On June 10, 2009, Zaldy Bugarin requested a loan from Northwest
Bank. The loan was approved for P100,000 at 10% bank discount for
75 days on June 13, 2009 . To bind the contract of loan, the following
bank discount note was issued by Zaldy:

14
The features of the note are the following:

1. The borrower/maker of the note is the party making the promise to pay (Zaldy
Bugarin).
2. The payee/lender of the note is the party to whom the promise is made
(Northwest Bank).
3. The face value/amount borrowed of the note is the maturity value of the loan
(P100,000)
4. The discount rate stated as an annual rate based on the face value of the note
(10%).
5. The term of the note is the length of time between the date of issue and when the
note matures (75 days).
6. The due date/date of maturity is the date on which the note is to be paid.
7. The face value is the amount payable on the due date of the note.
8. The proceeds of the loan is computed by subtracting the advance interest from
the face value.

The amount of bank discount on the promissory note could be computed easily
by using the Simple Discount Formula with some modifications; that is, the face
value, FV is substituted to the maturity value, MV in the formula. Thus, BD = MV
x r x t becomes BD = FV x r x t.

BD = FV x r x t
BD = 100,000 x 0.10 x 75/360
BD = 2,083.33

Solving for the proceeds:

P = FV – BD
P = 100,000 – 2083.33
P = 97, 916.67

Since discount is deducted, on the date the loan is obtained, Zaldy, the maker
of the note received only P97, 916.67 as proceeds of the loan. It is the amount he
applied for, the face value of the note that is due upon maturity.

EFFECTIVE RATE OF A BANK DISCOUNT NOTE

In a simple interest note, the borrower receives the full face value, whereas with a

15
bank discount note the borrower receives only the proceeds. Because proceeds are
less than the face value, the stated discount rate is not the true or effective rate of
the note.
Effective interest rate is computed using the formula:


Formula 5: Effective Interest Rate of a Bank Discount Note


EIR = BD ÷ [P x t]

where:

EIR effective interest rate
BD bank discount
P proceeds from the loan

t time

Example 15: What is the effective rate of a bank discount note for P200,000, at a
bank discount rate of 12% for a period of 5 months?

Solution: To find the effective interest rate, the bank discount and proceeds
should be known first. Hence, to compute:

a. the bank discount:

BD = FV x r x t
BD = 100,000 x 0.12 x 5/12
BD = 5,000

b. the proceeds:

P = FV - BD
P = 100,000 – 5,000
P = 95,000

c. the effective interest rate:

EIR = BD ÷ [P x t]
EIR = 5000 ÷ [95,000 x 5/12]
EIR = 0.12632 or 12.63%

Hence, the true or effective interest rate is 12.63%.

16
DISCOUNTING NOTES BEFORE MATURITY

In usual business practice, a business requires its lenders promissory notes to


bind the indebtedness between them. A business usually accepts notes, usually
short-term as payments for goods and services. However, there are also instances
when the payee opts to sell the notes to a third party, usually a bank or a financial
institution.

For example, the payee needs immediate cash to finance operations; he may sell
his notes receivable to have cash. It is obvious that the responsibility of collection,
in case of absolute sale will be vested upon the third party. In financial accounting
parlance, it is called receivable financing. This process in a more limited sense
is referred to as discounting of notes. Moreover, since the notes were sold, the
original bearer of the notes will then expect a lesser amount of proceeds compared
to what he could have, had he waited for the notes to mature.

When a note is discounted at a bank, the original payee receives the proceeds of
the discounted note while the new payee receives the maturity value of the note.
The time used to compute the proceeds is from the date the note is discounted up
to the maturity date. This is known as the discount period.

DISCOUNTING A SIMPLE INTEREST NOTE

Example 16: Mars Co. received P300,000 simple interest note which bears 6%
interest for 6 months from one of its customers. After 3 months, due
to financial difficulty, Mars Co. discounted the note at Banco de Oro
at a discount rate of 7%. Compute for the proceeds Mars Co. received
from the sale of the note.

Solution: a. Solve for the maturity value of the simple interest note:

MV = P (1 + rt)
MV = 300,000 [1 + (0.06 x 6/12)]
MV = 309,000

b. Count the unexpired portion of the term---the discount period. In


this case, 3 months had expired, therefore only 3 months are left.

c. Solve for the bank discount.

BD = MV x r x t
BD = 309 x 0.07 x 3/12
BD = 5,407.5

17
d. Solve for the proceeds.

P = MV – BD
P = 309,000 – 5,407.5
P = 303, 592.5

Thus, Mars Co. received P303, 592.5 as proceeds from the discounted note. Take
note that he received a lesser amount compared to the expected maturity value of
P309,000. The difference could be regarded as compensation for the bank buying
the note. After the maturity date, the customer would pay to BDO the amount of
P309,000.

DISCOUNTING A BANK DISCOUNT NOTE

Example 17: Miles Co. received a bank discount note from a customer with face
value of P50,000 for 5 months. After a month, the note was discounted
at Allied Bank at a discount rate of 9%. What are the proceeds Miles
Co. will receive from discounting of notes?

Solution: a. Count the unexpired portion of the term---the discount period. In


this case, 4 months were left.

b. Solve for the discount.



BD = MV x r x t
BD = 50,000 x 0.09 x 4/12
BD = 1,500

c. Solve for the proceeds.

P = MV – BD
P = 50,000 – 1,500
P = 48,500

Hence, Miles Co. received P48,500 from the discounted note.

The procedures involved in discounting a non-interest bearing note a similar to


those of a bank discount note. The face value of the non-interest bearing note is the
same as its maturity value, hence, same procedures are followed.

18
REVIEW QUESTIONS

1. What is interest?

2. Why is interest called money’s rent?

3. What are the two main types of interest?

4. Explain how simple interest is applied in business practice.

5. State the simple interest formula.

6. Explain the concept of time with respect to simple interest.

7. What are the four time combinations?

8. Discuss why Banker’s Rule is preferred in business practice?

9. What is a bank discount?

10. How does bank discount apply in business?

11. Define promissory note.

12. What are the elements of a promissory note?

13. Define and explain discounting of notes?

14. What procedures are observed in discounting notes?

15. Explain why the effective rate is different from the nominal rate when notes
are discounted?

19
PROBLEM 1. Simple Interest, Maturity Value and Manipulating the Simple
Interest Formula

1. Fill in the missing values.

I P R t
5,000 12% 5 years
1,000 40,000 8%
3,000 70,000 267 days
65,000 9% 3.75 years

2. Hershey paid an interest of P350 five months after borrowing a certain amount.
How much did she borrow if the simple interest rate is 5.75%

3. Find the simple interest on a P80,000 loan at 14.25% for 2 years and 3 months.

4. Russell invested P3,000 in fund with interest rate of 3.75%. How much interest
would she earn after 10 months?

5. Venus invested in a fund which pays 12% per annum. The amount of investment
is P50,000. How long would it take for the investment to double? How much is
the total interest earned after the term?

6. An interest of P7, 000 was earned in an investment of P60,000 for 3 years. What
was the agreed interest rate?

20
7. For borrowing P20,000 for 3 years and seven months, a lender charges a borrower
P1,050. What simple interest rate does the lender used?

8. It takes 5 years for an investment to double. If the investment is compensated


with a simple interest of P6,000, at what rate the money was invested?

9. Interest of P2,000 is earned after 6 years. The investment’s simple interest rate is
8.25%. How much is the principal amount?

10. Hubert borrowed P60,000 from Jessica. It bears 6% simple interest per annum
for 3.5 years. After the term, what would Hubert’s settlement with Jessica? What
amount of income would Jessica recognize for the first year?

PROBLEM 2. Concept of Time---Actual and Approximate

1. Find the time, in days, of each of the following notes using actual and approximate
time.

21
Approximate Actual
Date Remarks
time time
January 24, 2008-February 15, 2008 is a
2009 leap year
August 16, 2007-December
------
16, 2007
Year Y+1 is
May 03, Y-May 01, Y+1
a leap year
January 04, Y-February 09, Year Y is a
Y+1 leap year
April 12, Y-July 15, Y+2 ------

2. Find the interest on P80,000 investment at 9% interest from July 29 – December

3 of the same year using the four time combinations.

3. Find the interest on P20,000 worth of investment at 10% for 267 days.

4. Using the four time combinations, find the interest on P10,500 from October 13
to March 10 of the following year (a leap year) at 6 ¼% simple interest.

5. Find the interest on P5,980 at 8.75% of simple interest from September 12, Y to
May 24, Y+1 using approximate time.

6. Diwane borrows P8,000 on September 1, 2008 and promises to pay the principal
with an exact simple interest of 9.3% on March 12, 2009. What amount does Diwane

22
pay to discharge his debt at the end of the term if actual time is considered? 2009
is not a leap year.

7. Zanjoe borrowed P9,850 on April 22, 2010. He repaid his debt with simple
interest of 7.25 % on November 5, 2011 (not a leap year). How much did he pay
at maturity date? Use approximate time.

8. Refer to Problem 7, would your answer change had you used actual time? By
how much will it increase/decrease?

9. Five thousand pesos is due on February 20, 2008 (a leap year). It is agreed that
the debt will be settled on August 26, 2009 with exact simple interest of 14%
charged after the original due date. Using approximate time, find the amount
that must be paid on August 26, 2009.

10. Refer to Problem 9, find the amount that must be paid on August 26, 2009 using
exact time.

PROBLEM 3. Bank Discount

1. Determine the bank discount if one borrows P30,000 at 12% simple discount for
3 years and five months.

23
2. Discount P9,800 for 1 year and 9 months at 8 ¾% simple discount.

3. If P2,500 is the present value of P5,000 due at the end of 13 months, what is the
bank discount rate?

4. The discount on P2,987.5 is P500 at 9%. When is the amount due?

5. When should P9,720 be due if the present value is P9,000 at a simple discount
rate 5 ¼%?

6. What amount should be borrowed for 7 months at 9% simple discount if P4,000


is needed now?

7. Find the bank discount at the rate of 7% and the present value of a debt of P7,800
which is due at the end of 6 months.

24
8. How much should Hayden borrow for 3 years at 12% interest in advance if he
needs P8,000 to buy an appliance?

9. Mr. Yang needs P12,500 for the purchase of machinery. He is thinking of applying
for a loan at PNB. Should the bank offer him an amount with advance interest
of 12% for 2 years, what amount of loan would he have to apply for to get the
desired amount.

10. Niña Cadatal-Lazaro signed a P500,000 bank discount note at the Banco de
Oro. The discount rate is 13.5% and the term of the note is 11 months. What is
the amount of bank discount? What are Niña’s proceeds of the loan?

PROBLEM 4. Discounting of Promissory Notes

1. Mr. Hwang has a note dated August 15, 2008 which calls for a payment of P5,674.5
on July 26, 2009. On October 15, 2008, the note is discounted at 11% exact simple
interest. How much cash did Mr. Hwang get as proceeds of discounting his
note?

2. On March 7, 2011, Budoy Maniego borrowed P200,000, cash from Ana Manalastas
carrying a 12% simple interest to be paid on July 5, 2011. On April 26, 2011, Ana
Manalastas decided to discount the note at Philippine National Bank because
she needed money for her travel to Singapore. The note was discounted at the
bank with a discount rate of 15%.

a. The term of the note is ___________________


b. The discount period is ___________________
c. The maturity value of the note is ___________________

25
d. The proceeds from PNB amounted to ___________________
e. The effective interest rate is ___________________

3. You own BSA4 Enterprise and you received the following promissory note.

On April 20, 2011 you experienced financial distress and you decided to discount
your note at ChinaBank at 14% discount rate.

a. Determine the elements of the note:

1. Maker _______________
2. Payee _______________
3. Term _______________
4. Discount period _______________

b. Solve for the bank discount.

c. How much would you be taking home as proceeds from discounting the note?

d. Determine the effective rate.

4. Jollibee receives P7,345.32 for discounting a 120-day note 40 days before it


matures. Find the rate of the note in simple interest if its face value is P7,000 and
the bank discount is P105.35

26
5. Jimmy received a promissory note from Jonathan as payment for the services
rendered by him amounting to P9,670.50 dated June 24, 2008 and due on April
28, 2009 with simple interest at 16%. Jimmy could no longer wait for the note
to mature because he needs cash so he decided to have the note discounted at
15.2% on January 28, 2009. How much are the proceeds?

6. PR Bank charges 16% simple discount on short-term loans. Find the face value
of the note given to the bank if the Mr. Jung receives P5,000 on January 15, 2009
and the maturity date of the note is July 20, 2010.

7. Rosalinda draws a 150-day note to the order of Fernando Jose. Fernando Jose
wishes to obtain P5,600 for discounting the note immediately at a discount rate
of 11%. What should the face of the note be?

8. The face of a 9-month note with interest of 13.65% is P8,650. If the note is sold 4
months before its maturity at 12%, what should the proceeds be?

9. On February 18, 2008 (a leap year), Levie Grace draws a note to the order of
Daniel promising to pay P5,090 at the end of 9 months. What proceeds does
Daniel receive if he discounts the note at 11.36% on September 21, 2009.

10. On September 11, 2008, Paul John draws a note promising to pay Krystel
P3,684.50 with simple interest of 13% 300 days later. How much should the
creditor receive if he sells the note on February 3, 2009 at 14% exact simple
discount?

27
CHAPTER II

COMPOUND AMOUNT
LEARNING OBJECTIVES

• Gain an understanding on the Time Value of Money concept.


• Gain an understanding on the concept of compound interest.
• Compare the difference between simple and compound interest.
• Acquire basic knowledge on how to compute for compound interest.
• Understand the concepts of nominal, periodic and effective rate of interest.
• Acquire knowledge on time approximation for fractional interest periods.
• Understand the available techniques in solving other unknown variables in the
Compound Amount Formula.
Assume that an investor lends an amount of P1,000. Assume further that it bears a
simple interest of 10% per annum. In this case, the interest-bearing balance remains
at P1,000 at each given time period and each year’s interest payment is P100. We
could conclude therefore that with respect to the original principal and time, the
interest earned at a given rate is constant. In this chapter, we will be dealing with
interest computation that is not constant for a given period.

Some investors prefer to include the interest earned periodically to the interest-
bearing balance to accumulate more interest. It is obvious that when the interest
earned on the first year becomes a part of the original principal, the interest for the
second year then becomes higher, hence interest is not constant. Needless to say, the
interest is higher in this case because two bases are used for interest computation---
the original principal and the interest earned periodically. This reinvestment of the
interest to gain more interest is called compounding. Hence, the interest earned
on the investment is called compound interest. To fully understand the concept
of the succeeding investment topics, it is important that we have a background on
the concept of time value of money.

THE TIME VALUE OF MONEY

Suppose you have a choice to receive P10,000


now or at some later date. Assume further
that the second option is realizable or has a
high probability of happening. An investor in
this case would normally choose to receive the
money now. An investor would normally do because a peso now is worth more
than a peso to be received in one year, or at any later date. This statement sums up
the important principle that money has a time value.

Many economic decisions involve investing money now in the hope of receiving
more money later on. The reason that a peso now is worth more than a peso to
be received in the future is that you could invest the money now and have more
money than a peso at a later date. It does not matter whether you expect inflation
or deflation to change the purchasing power of money; you always prefer money
now to the promise of the same amount of money later. Investments involving
simple interest present an easy decision-analysis because the time value of money
concept is ignored. Why?

Remember that in simple interest, the interest computation depends solely on


the original principal. Thus, the interest earned periodically is ignored, although
with respect to the time value of money concept, it should also form part of the
interest-bearing balance. Because the fact that you invested an amount, all the
benefits/profits that you should have been enjoying had you invested it in other
investment alternatives should be compensated by interest. This investment view
reflects compound interest.

30
COMPOUND INTEREST

If, at stated period during the term of investment, the interest due is added to the
principal and thereafter earns interest; the sum by which the original principal had
increased by the end of the term of the investment is called compound interest,
I. In other words, compound interest is the interest resulting from the periodic
addition of the simple interest to the principal. The word “compound” refers to
the process of interest being reinvested to earn additional interest. With compound
interest, the total investment of the principal and the interest earned to date is kept
invested at all times. At the end of the term, the total amount due which consists
of the original plus the compound interest is called compound amount or future
value, FV. The time interval between the successive conversions of interest to
principal is referred to as compounding period, n. The number of compounding
periods per year is called the conversion frequency or the compounding frequency,
f.

SIMPLE AND COMPOUND INTEREST COMPARED

Simple interest and compound interest produce the same accumulated amount
over one measurement period (n = 1). Over a longer period, compound interest
produces a larger accumulated value than simple interest while the opposite is
true over shorter period. Simple interest becomes progressively less favorable to
the investor as the period of investment increases.

Illustration: Find the final amount of a loan at the end of three years if the principal
or the face value is P20,000 and the interest rate is 10% compounded
annually. Compare the simple and compound amount as stated.

Simple Interest

The interest rate is applied only to the original principal amount in computing the
amount of interest.

Given that P = P20,000, t = 3 years and r = 10%

Period Principal Interest Ending Balance


1 20,000 2,000 22,000
2 20,000 2,000 24,000
3 20,000 2,000 26,000
Total P6,000

Compound Interest

The interest is applied to the original principal and any accumulated periodic
interest.

31
Given that P = P20,000, t = 3 years, r = 10%, n = 3 (n is computed as and t x f) and
f = 1 (annually).

Period, n Principal Interest Ending Balance


1 20,000 2,000 22,000
2 22,000 2,200 24,200
3 24,200 2,420 26,620
Total P6,620

This shows that the compound interest P6,620 is more than the simple interest
P6,000 by P620.

The illustration provides that the interest is compounded annually, meaning,


interest is earned at the principal and the accumulated amount of interest once a
year. There are times that the conversion frequency is more than once a year. The
most common conversion frequency can be summarized as follows:

Number of compounding
Conversion frequency, f
periods in a year, n
Annually 1
Semi-annually 2
Quarterly 4
Bi-monthly 6
Monthly 12

What happens when interest is compounded more than once a year? In the same
example, except that interest is compounded semi-annually:

Given that P = P20,000, t = 3 years, r = 10%, n = 6 and f = 2 (semi-annually)

Here, note that the time factor now is 6/12 or ½ or .50 because semi-annual
compounding means every six months. With regards to the interest rate, r, it
is expressed as periodic interest. The whole rate is divided by the conversion
frequency.
Illustration: Let us assume that a nominal rate of 10%. The periodic rate would
depend on how frequent is the compounding to be made. The
changes in the periodic interest with respect to conversion frequency
can be summarized below:

Number of compounding Periodic


Conversion frequency, f Interest, r/f
periods in a year, f
Annually 1 10%
Semi-annually 2 5%
Quarterly 4 2.5%
Bi-monthly 6 1.67%
Monthly 12 0.83%

32
When a problem calls for a compounding without specified frequency, it is
assumed that the conversion frequency is annually or done once a year.

Hence, in the example, r is expressed as 10%/2 because interest is compounded


twice a year. The compound interest schedule is presented below:

Period, Ending
Principal Interest
n Balance
1 20,000 1,000.00 21,000 Interest is computed from the original principal
2 21,000 1,050.00 22,050 I = 21,000 x .05; EB = 21,000 + 1,050
3 22,050 1,102.50 23,152.50 I = 22,050 x .05; EB = 22,050 + 1,102.50
4 23,152.50 1,157.63 24,310.13 I = 23,152.50 x .05; EB = 23,152.50 + 1,157.63
5 24,310.13 1,215.51 25,525.64 I = 24,310.13 x .05; EB = 24,310.13 + 1,215.51
6 25,525.64 1,276.28 26,801.92 I = 25,525.64 x .05; EB = 25,525.64 + 1,276.28
*EB means Ending Balance
Total P6,801.92

For the same investment variables, semi-annual compounding yields P181.92


more than annual compounding.

Interest at semi-annual compounding 6,801.92


Interest at annual compounding 6,620.00
Difference P181.92

Thus, it may be generalized that given the same investment information, more
compounding periods means more interest.

COMPOUND AMOUNT

Annual interest rate must be adjusted to reflect the period of compounding. Where
no conversion is stated in any investment problem, it is assumed that the interest
is compounded annually. Let P be the original principal and let FV be the future
value to which P accumulates by the end of n conversion period.

Figure II.1 Compound Amount

33
In reference to the timeline above, we can deduce that compound amount is
derived as:

Original P invested P
Interest due at the end of the first compounding period Pi
New principal at the end of the first compounding period P(1 + r)
Interest due at the end of the second compounding period P(1 + r)r
New principal at the end of the second compounding period P(1 + r)2
Interest due at the end of the third compounding period P(1 + r)2r
New principal at the end of the third compounding period P(1 + r)3
. .
. .
. .
By the end of the nth compounding period P(1 + r)n

To accumulate a principal P for n compounding periods means to find the


compound amount FV resulting at the end of n periods if P is invested at the
specified interest rate.

Manual calculation of compound amount is easy when the n is 2, 3 or maybe 4.


But how if the number of compounding periods becomes 50, 60 or even 100? To
address this problem, mathematicians have developed the Future Value formula.

Formula 1: Future Value



FV = P (1 + r/f)n


where:

FV future value

P principal/original amount/present value

r interest rate; expressed as periodic interest

n total number of compounding periods
f conversion frequency per year

Example 1: Mark Jordan invested P5,000 for 10 years at 8% compounded


quarterly; find the accumulated amount after the investment period.

34
Solution: Given that P = 5,000, t = 10 years, f = 4, r = 8% and n = 40

FV = P (1 + r/f)n

FV = 5,000(1.02)40

FV = 11,040.20

Accumulated amount can also be accumulated using the Time Value


of Money Factor Table; Table I---Future Value Interest Factor.

FV = 5,000(1.02)40

Since (1.02)40 = 2.208039664, then;
FV = 5,000(2.208039664)* *
amount rounded in Table I (2.208)
FV = 11,040.20

Example 2: Accumulate P8,000 for 15 years at 9% compounded semi-annually.

Solution: Given that P = 8,000, t = 15 years, f = 2, r = 6% and n = 30


FV = P (1 + r/f)n

FV = 8,000(1.045)30

FV = 29, 962.55

Example 3: Atasha invested P10,000 in an investment that yields 15% interest


compounded monthly for 5 years. Determine the compound amount
at the end of the term. Compute for the compound interest.

Solution: Given that P = 10,000, t = 5 years, f = 12, r = 15% and n = 60

a. To solve for the future value

FV = P (1 + r/f)n

FV = 10,000(1.0125)60

FV = 21,071.81

b. To solve for the Compound Interest



I = FV - P
I = 21,071.81 – 10,000
I = 11,071.81

Example 4: Find the amount due at the end of 13 months if P5,000 is invested at
16% compounded monthly.

Solution: Given that P = 5,000, t = 13/12 years, f = 12, r = 16% and n = 13

35
FV = P (1 + r/f)n

FV = 5,000(1 + 0.16/12)13

FV = 5,939.51

There are also instances when money is received at different time intervals and
we are faced with determining their single future value at a certain date. In such
cases, combined factors of Future Value are used. Let us try the example below.

Example 5: The following amounts are received at the end of year and invested
immediately at 10% compounded semi-annually: in Year 1, 500; in
Year 2, 600; Year 3, 700 and in Year 4, 800. At the end of year 4,
what is the total cash received assuming that the investments are all
collected at year 4 (maturity date).

Solution:
Investment 1: Investment 2:

FV = P (1 + r/f)n FV = P (1 + r/f)n
FV = 500(1 + 0.10/2)6 FV = 600 (1 + 0.10/2)4
FV = 670.05 FV = 729.30

Investment 3: Investment 4:

FV = P (1 + r/f)n FV = P (1 + r/f)n
FV = 700(1 + 0.10/2)2 FV = 800(1 + 0.10/2)0
FV = 771.75 FV = 800

No time to gain interest because the money is


invested at year-end.

Then FV­T = 670.05 + 729.30 + 771.75 + 800


FV­T = 2,971.10

Therefore, at the end of Year 4, you have a total amount of P2,971.10. That is, the
future value of the uneven cash flow.

Alternatively, FV­T = P (1 + r/f)6 + P (1 + r/f)4 + P (1 + r/f)2 + 300

FV­T = 500(1 + 0.10/2)6 + 600 (1 + 0.10/2)4 + FV = 700(1 + 0.10/2)2 + 300


FV­T = 2,971.10

FRACTIONAL PART OF COMPOUNDING PERIODS

Previous discussion of compound interest assumed that n is a whole part in


which compound interest could be computed with ease. Accumulated amount is

36
determined by the formula FV = P (1 + r/f)n even when n is not a whole part. For
example, when the compounding frequency is quarterly but the term of the loan
is 1 year and 2 months, the use of the Future Value formula could still be helpful.
However, when investment problems require separate computation of the whole
and the fractional part of the compound interest, following steps are suggested.

1. Find the accumulated value of the principal at compound interest up to


the last whole interest period.
2. Accumulate the resulting FV¬1 for the remaining time at simple interest
at the given nominal rate. Simply stated, using the resulting Future
Value, find the interest for the remaining period using the simple interest
formula.
3. Add the results of 1 and 2

Figure II.2. Fractional Time Periods

Illustration: Find the amount at the end of 2 years and 2 months if P1,000 is
invested at 10% compounded quarterly, using simple interest for
any time less than an interest period.

Solution: Given that P = 1,000, t = 2 + 2/12 years, f = 4, r = 10% and n = 8 for


the whole part

FV1 = P (1 + r/f)n

FV1 = 1,000(1.025)8

FV1 = 1,218.40

I = Prt

I = (1,218.40)(0.10)(2/12)
I = 20.31

FV­T = 1,218.40 + 20.31


FV­T = 1,238.71

37
UNKNOWN RATE OF INTEREST

Our previous discussion on manipulating the simple interest formula requires us


to have basic algebraic skills on formula derivation. Like in the case of Future
Value formula, solving for other variables that are unknown are equally important
because it gives us a total understanding on the concept of compounding.

We have two available methods for solving r. First, is through the use of Direct
Method, wherein we will apply the rules involving radicals and the second is
through the use of linear interpolation.
The first is solving r by directly using a scientific calculator. This works well if a
single payment is involved.

Example 6: At what interest rate compounded semi-annually would P20,000


accumulate to P30,000 in 6 years.

Solution: Using formula FV = P (1 + r/f)n then,



30,000 = 20,000(1 + r/2)12

Divide both sides by 20,000, what is left is:


(1 + r/2)12 = 1.5

Extract the 12th root of both sides to eliminate the radical sign, then:

1 + r/2= 1.034366083

Using algebraic operations; by transposition, we have



r/2 = 1.034366083– 1
r/2 = 0.034366083

By cross multiplication, we have

r = 2(0.034366083)
r = 0.068732166 x 100
r = 6.87% compounded quarterly
The second method is through linear interpolation. The value of r is derived from
interpolating two factors on which the missing unknown lies in between them.

Example 7: If P5,000 will become P15,564.8 after 10 years, what is the interest

38
rate compounded quarterly?

Solution: Using formula FV = P (1 + r/f)n then,


(1 + r/f)n = FV/P

Since it is compounded quarterly, we could express it as:

(1 + r)n = FV/P
(1 + r)40 = 15,564.80/5,000
(1 + r)40 = 3.11296

Scanning across the values of (1 + r)40, we could not locate the exact value of 3.11296.
Hence, we could find it between the factor values of 2% and 3% which are 2.208
and 3.262. From this difference, we could interpolate between the values with the
assumption that the difference in value factors would lead us to equal difference
between the rates that yields the interest rate for value factor 3.11296 .

Interest Table
Rate per factor or
period (1 + r)n
3% 3.262 (1)
r 3.11296 (2)
2% 2.208 (3)

(2) – (3) r – 2% = 0.90496 (4)


(1) – (3) 1% 1.054 (5)

By cross multiplication;

(1% x 0.90496) = (r – 2% )(1.054)


0.0090496 = 1.054 r – 0.02108

By transposition;

1.054 r = 0.0090496 + 0.02108


1.054 r = 0.0301296

Dividing both sides by 1.054, the value of r is:



r = 2.858595825

Since, we are looking for the whole nominal rate and given that the
compounding frequency is 4 (quarterly), we multiply the value of r by 4.
Hence;

r = 11.43%

39
UNKNOWN TIME

Like unknown interest rates, unknown time could also be


solved using different methods. This workbook will only
introduce two methods of solving unknown time. The first
method is through the use of logarithms. This method works
well if a scientific calculator is available. The other method is through the use of
linear interpolation as in the case of unknown rate of interest.

Example 8: Mr. Choi invested P12,000 at an interest rate of 12% compounded
semi-annually at a certain time period. If the investment accumulated
a maturity value of P30,563.90, how long is the agreed term of the
investment?

Solution: Given that P = 12,000, FV = P30,563.90, f = 2 and r = 12%

Using formula FV = P (1 + r/f)n then,



(1 + r/f)n = FV/P
(1.06)n = 30,563.9/12000
(1.06)n = 2.546991667

Since 2.546991667 = (1.06)n, from Table I of the appendix, (1.06)16 =


2.540 and (1.06)17 = 2.693. Thus, n is between 16 and 17.

By linear interpolation,

Compounding Table factor or
period (1 + 2%)n
17 2.692 (1)
n 2.546 (2)
16 2.540 (3)

(2) – (3) n –16 = 2.546 - 2.540 (4)


(1) – (3) 1 2.692 - 2.540 (5)

The value of n can now be solved from the proportion formed by the differences
on lines (4) and (5).

n – 16 = 0.006
0.152

n = 0.006 + 16
0.152

n = 16.04, number of semi-annual payments.

40
Thus, the required time is 8.02 years.

Alternatively, the use of logarithms could help us compute time problems


involving this type as follows:

Example 10: Shamcey invested P30,000 in the Miss Universe Bank. After a
certain time, her investment gained P5,650 interest. If the bank pays
12% compounded quarterly, how long was the term of Shamcey’s
investment?

Solution: Given that P = 30,000, FV = P35,650, f = 4 and r = 12%

By logarithm,

FV = P (1 + r/f)n

35,650 = 30,000(1.03)n

Dividing both sides by 30,000;


1.188333333 = (1.03)n

log 1.188333333 = log (1.03)n


log 1.188333333 = n log (1.03) Note: log ax = xlog a

log 1.188333333 = n log (1.03)


log 1.03 log 1.03
n = log 1.188333333
log 1.03
n = 5.84

Thus, the time required is 5.84/4 which is approximately 1.46 years.

NOMINAL, PERIODIC AND EFFECTIVE RATES

The term “effective” is used for rates of simple interest (and simple discount) in
which interest is paid once per measurement period (usually a year), either t the
end of the period or at the beginning of the period, as the case may be. When
interest is paid more frequently than once per measurement period (other than a
year), it is called nominal rate of interest (or discount). That is,

Nominal rate Nominal means “in name only”. This is sometimes called
the quoted rate.
Periodic rate The amount of interest you are charged each period.
Effective annual rate The rate you are actually get charged on an annual basis

Illustration: Lender A might charge an annual effective rate of 8% on loans while

41
lender B might charge 8% compounded quarterly.

Analysis: Lender A is charging an effective rate while lender B is charging


what we call a nominal rate of interest of 8% compounded quarterly.
Eight percent (8%) compounded quarterly is equivalent to saying
2% per quarter which is called the periodic rate.

Example 11: Let us say a credit card has an interest rate of 3% per month. What is
the annual rate you are actually charged?

Solution: The nominal rate is 36% compounded monthly


. The periodic rate is 3% per month
The effective annual rate is 42.57%

When interest is compounded annually, the effective rate is equal to the nominal
rate but when it is compounded more than once a year, effective rate is greater
than the nominal rate. When comparing different rates compounded at different
conversion periods per year, compute their respective effective rates.

Formula 2: Effective Rate



r eff = [(1 + r nom/f)f ]- 1

where:

r eff effective rate

r nom nominal rate
f conversion frequency per year

From example 11,

r eff = [(1 + r nom/f)f ]- 1


r eff = [(1 + 0.36/12)12]- 1
r eff = (1.03)12 -1
r eff = 0.4257 x 100
r eff = 42.57%

Example 12: What nominal rate converted monthly yields the effective rate of
5%?

Solution: Given that f = 12 and reff = .05


reff = [(1 + r nom/f)f ]- 1
0.05 = [(1 + r nom/12)12] – 1

42
By transposition, we have:

1.05 = [(1 + r nom/12)12]

By extracting the 12th root of both sides, we have:

(1.05)1/12 = 1 + r nom/12
(1.05)1/12 - 1 = r nom/12
12[(1.05)1/12 – 1] = r nom
r nom = 0.0489 x 100
r nom = 4.89 compounded monthly

VARYING INTEREST RATES

If the interest rate changes as the principal is being compounded, the compound
amount is the product of the principal and the several accumulation factors, which
are at different interest rates for specific given periods.

Illustration: If the principal of P10,000 and the interest rate is 8% compounded


semi-annually for the first three years, and 6% compounded annually
for the next 3 periods. What is the accumulated amount after 6 years?

Solution: a. Solve for the FV for the first three years. Thus, given that P = 10,000,
t = 3years, f = 2, r = 8% and n = 6.

The future value is computed using the FV formula:

FV = P (1 + r/f)n

FV = 10,000(1.04)6

FV = 12,653.19

b. Solve for the future value for the next 3 years after the first
accumulation. Thus, the future value of P12,653 after the first
three years becomes the principal for another 3 years.

Hence;

FV = P (1 + r/f)n

FV = 12,653.19(1.06)3

FV = 15,070.15

The total compound amount after 6 years is P15,070.15 and the


accumulated compound interest is P5,070.15.
REVIEW QUESTIONS

43
1. What is compound interest?

2. How does compound interest differ with that of the compound interest?

3. Which is more favorable among simple and compound interest from a creditor’s
point of view? How about from the debtor’s?

4. State the Future Value formula.

5. Differentiate nominal, periodic and effective rates of interest.

6. How do we approximate accumulated vale for fractional interest period?

7. What are the techniques available to solve unknown rate of interest?

8. What are the techniques available to solve unknown time?

9. How is interest per period determined?

44
10. Explain the time value of money concept.

PROBLEM 1. Future Value and Compound Interest

1. Fill in the blank items.

Principal Annual Term of Interest Future Compound


Rate Loan Compounded Value Interest
1. P50,000 15% 4 years Bi-monthly
2. P25,000 8% 10 years Quarterly
3. P56,000 12% 6 year Annually
4. P23,000 10% 5 years Semi-annually
5. P1,000 13% 3 years Quarterly

2. Convert the given nominal interest rate to periodic interest and the total number
of interest periods.

Nominal Term of Interest Periodic Number


Rate Loan Compounded rate of interest
periods
1. 12% 5 years Annually
2. 9% 8 years Semi-annually
3. 16% 4 years Quarterly
4. 10% 6 years Bi-monthly
5. 8% 3 years Monthly

3. Compute for the missing items.

Nominal Term of Interest Periodic Number of


Rate Loan Compounded rate interest periods
1. 12% 1% 72
2. 9% 2.25% 36
3. 16% 8% 20
4. 10% 2.5% 60
5. 8% 8% 20

4. Judy Abott borrows P5,000 at 14.8% compounded monthly. How much should

45
she pay at the end of 2 years and 4 months to settle her debt?

5. Accumulate P6,432.73 at 16% compounded semi-annually for 9 years and 6


months.

6. What is the simple interest accrued if P5,000 is invested at 8% for 5 years? How
much compound interest would be accrued if it’s invested otherwise at 8%
compounded quarterly for 5 years? Obtain the difference.

7. Bryan Dave invested P5,000 in a fund that pays 8% compounded quarterly for 2
years. Prepare a schedule of periodic increase in principal and interest.

8. Maria Nina deposits P10,520 in a bank that pays interest at 11.45% compounded
quarterly. How much should be in her account in 4 years if in 3 ½ years she
withdraws P5,000?

9. What is the compound amount and compound interest of P200,000 compounded


semi-annually for 4 years invested at 10%?

10. You are asked to analyze an investment decision.

46
Option A Option B Option C
Principal, P P100,000 P100,000 P100,000
Interest, I 12% 12% 12%
compounded compounded compounded
quarterly semi- bi-monthly
annually
Term, t 5 years 5 years 5 years

Rank the preceding investment options according to desirability. What criteria


have you used? Justify your evaluation.

PROBLEM 2.Compound Amount

1. Find the accumulated value and compound interest accumulated of the following
if the present value is P1,100 at 6%.

a. compounded annually for 7 years.

b. compounded semi-annually for 10 years.

c. converted quarterly for 16 years.

d. converted monthly for 15 years.

2. How much would be accumulated if Charry invested P12,000 for 4 years at 9.5%
compounded semi-annually.

47
3. Accumulate P5,000 for 20 year at 8.5% compounded quarterly.

4. How much must be paid on the due date June Y+1 on a loan of P35,000 made on
December Y with interest rate of 5% convertible semi-annually?

5. Jayron deposits P13,890 in a bank account paying 5% compounded monthly.


How will he save at the end of 15 years?

6. As a 50th birthday present to his wife, a husband deposited P16,800 in her account
in an investment that pays 5% converted quarterly. How much would she have
in the fund on her 66th birthday?

7. A child receives a gift P20,000 deposited in his account on his 18th birthday. If the
bank pays 9.25% interest compounded monthly and no withdrawals are made,
how much should be credited in his account on his 21st birthday?

8. Accumulate P10,440 for 6 ½ years at 12.5% compounded bi-monthly.

48
9. To create a fund, P9,000 is deposited in the bank. If interest is 9% compounded
quarterly for 3 years, how much would be the investment’s future value?

10. Darwin borrows P3,000 with interest at 12% compounded quarterly. How
much should he pay to the creditor after 4 years to pay off his debt?

PROBLEM 3.Unknown Rate of Interest

1. What must be the rate of interest compounded semi-annually for an amount to


double in 5 years and 6 months?

2. If P2,500 is to be accumulated to P5,000 at the indicated time, find the effective


rate if t =

a. 10 years

b. 9 years

49
c. 2 years

3. If P500 is to be accumulated to P1,500 at the indicated time, find the effective


rate if t =

a. 5 years

b. 2 years

c. 1 year

4. If P1,000 is to be accumulated to P2,000 in 5 years, what is the rate compounded

a. quarterly

b. semi-annually

c. annually

5. If P4,000 is to be accumulated to P12,000 in 5 years, what is the rate compounded

50
a. quarterly

b. semi-annually

c. monthly

6. Find the effective rate of interest if P2,250 is the present value of P5,250 which is
due at the end of 6 years.

7. Find the effective rate of interest if P5,000 is the present value of P15,000 which
is due at the end of 9 years.

8. P27,980 is the future value of an amount of P20,000 invested now at a certain


rate for 3 years compounded quarterly. At what interest rate is this yield possible?

9. P80,000 will become P87,650 at the end of 5 years if invested at a certain rate
compounded semi-annually. Find the interest rate.

10. An amount of P16,000 will be doubled after 5 years if invested at an interest

51
rate compounded semi-annually. Determine the interest rate used.

PROBLEM 4. Unknown Time (Use Both Interpolation and Direct Method)

Principal Interest Rate Frequency of Compounding Maturity


Value Compound Interest Number of periods
a. 22,000 12% Semi-annually 31,000
b. 20,000 11% Annually 5,400
c. 60,000 9% Monthly 69,600
d. 4,500 8% Quarterly 8,000
e. 58,800 10% Monthly 4,900

1. For a certain time, an investment of P45,000 accumulates to P56,800 if invested


at 12% compounded semi-annually. Compute for the term of the investment.

2. Rosevie invested in a fund that pays 9% compounded quarterly. If the invest-


ment accumulates from P55,000 to P62,750, determine the term required for the
investment to yield such return.

3. Conrad determined that the future value of his P25,000 is P32,890. If he is


willing to receive an interest rate of 10% compounded annually, compute for the
term of his investment.

4. How long will it take P45,000 to accumulate to P51,500 if invested at 11% com-
pounded monthly?

5. Britney wants to receive a compound interest of P5,760 for his investment of


P15,000. If he is paid an interest of 8% compounded semi-annually, compute for
the time required for the investment.

6. Andi invested P25,000 in a special account that will gain 10% compounded
semi-annually after a certain time. If the investment will accumulate to P28,543,

52
determine the term of the investment.

7. Richard deposited P30,000 in a special account bearing interest rate of 12%


compounded semi-annually. If the amount will become P37,900, compute for the
time required to accumulate such amount.

8. Severino deposited the amount of P25,000 in a special account with interest


rate of 9% compounded quarterly. If he was able to negotiate a total return of
investment plus interest of P29,840, determine the time required for the invest-
ment.

9. Talah expects that her P90,000 investment will earn a compound interest of
P12,54o after a certain time if invested at 12% compounded semi-annually. How
long is the term of the investment?

10. How much time is required for a P10,000 worth of investment accumulate to
P15,870 if invested at 9% compounded quarterly?

PROBLEM 5. Varying Interest Rates, Nominal, Periodic and Effective Interest


Rates.

1. Valentino invested an amount of P22,000. The term of the investment is 5 years


but the interest rate varies. For the first 2 years, it will earn 8% compounded
quarterly and from the third through the fifth year, 9% compounded semi-annu-
ally. Compute for the total future value.

2. Richie Anne invested P30,000 in a special account. The investment will last for
5 years, however, the interest rate varies. For the first two years, it will earn a

53
simple interest of 12% per annum. For the remaining term, a compound interest
of 9% compounded semi-annually.

3. Chiranjibi invested P9,000 for a year at 12% compounded quarterly, find the
effective rate of interest.

4. If an amount of P35,000 is invested for 3 years at 9%. Find the effective rate if
the interest is compounded:
a. Annually

b. Semi-annually

c. Quarterly

d. Monthly

e. Bi-monthly

5. If an amount is invested at 12% compounded quarterly for 4 years, determine


the following:
a. Nominal rate

b. Periodic rate

c. Effective rate

6. What is the equivalent effective rate of 10% compounded semi-annually.

7. An amount is invested at 8% compounded monthly, compute for its equivalent


effective rate.
a. Assume the conversion frequency is once a year, what is its effective rate?

54
8. Mr. Golda invested a certain amount at 12% compounded monthly. Because of
lack of knowledge in investments, he wants to determine the following:
a. The nominal rate

b. The periodic rate

c. The true rate of investing

9. Decide which is a better investment:


Investment I. 12% compounded quarterly; 3 years

Investment II. 10% compounded monthly; 3 years

10. Bellefire wants an effective rate of 10.38%. If the compounding frequency is


four times a year, what is the nominal rate? What is the periodic rate?

55 55
CHAPTER III

PRESENT VALUE
LEARNING OBJECTIVES

• Gain understanding on the concept of present value.


• Understand how does the present value concept apply to business practice.
• Acquire basic knowledge on how to compute for the present value of an
investment that involves single receipt or payment (lump sum).
• Understand discounting with fractional discounting periods.
• Acquire knowledge about equations of values of a set of obligations.
We have learned from our previous discussion the concept of future value or
accumulated amount. The objective of compounding is to find the final amount
due of principal, P when invested, at the end of time, t at certain interest rate, r.

There are times when it would be useful to know the value of certain amount or
investment due in the future. That is, the future value is stated on present value.
Synonymously, when future value is stated on present value, an investor would
simply want to know, how much would be invested now (time 0) to accumulate a
certain future value.

PRESENT VALUE

The present value, PV of a certain amount due in the future (future value) is the
amount or principal, P that must be invested now that will grow to earn an interest,
I at certain interest rate, r for a certain time period, t. The process of finding the
present value of a future value of an investment is called discounting. Take note
that the term “discounting” in this chapter should not be interchangeably used as
in the case of discounting promissory notes.

To discount an accumulated amount or future value, FV for compounding periods,


n at periodic rate, r means to find the present value originally invested to arrive at
a certain future value. To illustrate such process, we could use a timeline.

Figure III.1 Present Value


In order to compute the present value of a certain amount accumulated in the
future, we use the following formula:

58
Formula 1: Present Value


PV = FV (1 + r/f)-n or

PV = FV
(1 + r/f)n


where:

FV future value

PV principal/original amount/present value

r interest rate; expressed as periodic interest

n total number of compounding periods
f conversion frequency per year

Example 1: Beryl wants P16,000 compound amount of her investment after


3 years. The investment bears an interest of 12% compounded
quarterly. How much would she invest now to expect the desired
amount after the term?

Solution: Given that FV = 16,000, r = 12%, t = 3 years and f = 4; using Formula


1:

PV = FV (1 + r/f)-n
PV = 16,000(1 + 0.12/4)-12
PV = 11,222.08

Alternatively,

PV = FV
(1 + r/f)n

PV = 16,000
(1 + 0.12/4)12

59
PV = 11,222.08

Or using the Present Value Factor in Table II, we have:

PV = FV (1 + r/f)-n

Since the value of (1.025) -12 is 0.701 (rounded value), we have

PV = 16,000(0.701)
PV = 11,216* *difference due to rounding of values

Example 2: An investment with an amount due to be received in the future of


P25,500 bears an interest rate of 8% compounded semi-annually for
5 years. Find its present value. Obtain the amount of discount.

Solution: PV = FV (1 + r/f)-n
PV = 25,500(1 + 0.08/2)-10
PV = 16,889.10

Since the process of finding the present value of an investment is


called discounting, the discount referred here into is simply the
amount of compound interest. The discount/compound interest is
obtained by subtracting the present value from the present value of
the investment.

I = FV – PV
I = 25,500 – 16,889.10
I = 8,610.90

FRACTIONAL PART OF DISCOUNTING PERIODS

Some investment problems require the computation of present value which


involve fractional time periods. When faced with this type of
investment problems, the following steps are suggested:

1. Discount the future value for the whole conversion


period and add 1. In this manual, we call the resulting
value as PV1.
2. Accumulate PV1 using simple interest on the time
difference between the whole compounding period
and the term of the investment.

60
3. Compute for the final present value. This is obtained by adding the result
from step 1 and step 2. Compound interest is obtained by finding the
difference between the compound amount and the resulting present
value from the computation.

Example 3: The future value of an investment is worth P20,000. Discount the


compound amount for 5 years and 7 months at 10% compounded
semi-annually.

Solution: Given that t = 5 + 7/12 years, FV = 20,000, f = 2, r = 10% and n = 12



PV1 = (20,000)(1.05)-12
PV1 = 11,136.75


I = (11,136.75)(0.05)(5/12) (6 years – 5 and 7/12 years)
I = 232.02

PVT = 11,136.75 + 232.02


PVT = 11,368.77

EQUATIONS OF VALUES

Investments made by creditors to their debtors are really intended to reach the
maturity date for the mutual benefit of the money’s usage (debtor) and interest
compensation (creditor).

However, there are instances when an obligation or a set of obligations maybe


replaced by a single obligation or set of obligations on a time that is different
from the maturity date set on the investment contract. This maybe a result of debt
restructuring, that is changing the terms of the contract of debt or obligations.

In such instance, it is important that investors and debtors have the knowledge on
how to obtain the equivalent values of the set of obligations should the settlement
is made before or after the maturity date originally agreed by them.

In the process of determining the values equivalent to the original obligations,


there is a so-called known date which serves as a reference point in the comparison.
The process of determining the equivalent values of investments depends on the
comparison date. If the comparison date or known-date is before the maturity
date, discounting is used. It simply calls for the computation of present value of a
known amount of obligation at a known date. On the other hand, if the maturity
date falls before the comparison date/known date, accumulation is used. In this
case, an amount of obligation is brought to future time which simply the known
date.

61
In problems involving equations of values, it is important for a learner to
understand that the old obligations are replaced by new ones. For example, when
a set of obligation was agreed to be paid in installment; the payment in installment
becomes the new obligation replacing the set of old ones. There are times that the
payment date is not the comparison date called for the problem. But since, this is
equation of values, whether the comparison date coincide with the payment date
is not an issue because it will yield the same amount (see example).

In the settlement of a set of obligations, the settlement rate is the prevailing interest
rate and not the rate originally attached with the loan, except when the rate is
constant.

Actually, there are many approaches to solving equations of values. For us to


understand the concept of determining equivalent values of obligations, let us
illustrate it using a timeline.

Figure III.2 Equations of Values-Discounting

In the above timeline, values which are stated on their maturity value are brought to
the comparison date (payment date may be different) by the process of discounting
because they lie after the comparison date. If the payment date (new obligation)
lies after the comparison date, we also have to discount it.

62
Figure III.3 Equations of Values-Accumulating

In the above timeline, values which are stated on their maturity value are brought
to the comparison date (payment date may be different) by the process of
accumulating because they lie after the comparison date. If the payment date (new
obligation) lies before after the comparison date, we also have to accumulate it. It
is to be noted that in the equation of values, the following must be equated and
should both stated on values at comparison date:

New obligation (payment) = Old Obligations

Example 4: Kim Chiu owes Gerald the following set of obligations:

Debts Due after


Debt 1:P3,000 2 years
Debt 2:P5,000, 8% simple interest 3 years
Debt 3:P9,000, 8% compounded quarterly 6 years

If the parties agree that a single payment is to be made after 5 years,


determine the amount of payment if money is worth 9% (settlement
rate) compounded semi-annually.

Solution 1: Comparison Date is the Payment Date-Year 5

The first step is for us to compute for the maturity values of the
obligations.

Debt 1: P3,000 is already the maturity value due after 2 years.


Debt 2: MV = 5,000 [1 + (0.08 x 3 years)] = 6,200
Debt 3: MV = 9,000(1 + 0.08/4)24 = 14,475.94

63
The next step is to plot them to a timeline for us to know the process
to be used whether to discount or accumulate the values.

Debts 1, 2 and 3 which are the old obligations as seen in the table
are expressed at their maturity values. They are to be replaced by
the payment to be made at Year 5 (which is this case is Debt 4), the
payment date which is also our comparison date. We name the new
obligation (payment) or simply the equivalent value as x. Using our
formula, we have;

New Obligation = Old Obligations


x = 3,000(1 + 0.09/2)6 + 6,200(1 + 0.09/2)4 + 14,475.94(1 +
0.09/2)-2
x = 3,906.78 + 7,393.61 +13,256.05
x = 24,556.44

Debts 1 and 2 are accumulated because they lie before the comparison
date. Debt 3 lies after the comparison date; therefore we discounted
it by 2 periods. Debt 4, the new obligation is not discounted nor
accumulated because the comparison date is also the payment date.
This means that at Year 5, the old obligations is replaced by the
amount to be paid by Kim Chiu at Year 5 amounting to 24,556.44.

Solution 2: Payment Date and Comparison Date are not the same. Payment Date =
Year 5; Comparison Date = Year 4

Same procedures are to be followed except that we bring the Old


Obligations as well as the New Obligation at Year 4, the comparison
date. Using a timeline:

64
Using our formula, we have;

New Obligation = Old Obligations


(1 + 0.09/2)-2x = 3,000(1 + 0.09/2)4 + 6,200(1 + 0.09/2)2+
14,475.94(1 + 0.09/2)-4
0.915729951x = 3,577.56 + 6,770.56 +12,138.96
0.915729951x = 22,487.08

Dividing both sides by 0.915729951, we have;

x = 24,556.45* *.01 difference due to rounding

This means that Kim Chiu will pay Gerald the amount of 24,556.45
at Year 5. Learners should not be confused with Year 4, it served
only as comparison date.

Example 5: Romar Angelo borrowed an amount from Alfonso Tomas. The


amount borrowed has a maturity value of P80,000 after 4 years.
Because the amount borrowed was not sufficient, Romar Angelo
borrowed another amount from Alfonso Tomas with maturity value
of P50,000 after 5 years. If money is worth 5% compounded semi-
annually, what single amount of obligation would Romar Angelo
have to pay Alfonso Tomas for the settlement of the debts at Year 3?
Comparison date = Payment date.

Solution: Let us first illustrate it using a timeline:

65
The problem simply asks for the single amount that would settle the
two obligations at Year 3, one and two years earlier than the maturity
dates of the two obligations respectively. Therefore, to determine
the equivalent value, we will discount the maturity value of the two
obligations at Year 3. Hence; with EV = Equivalent Value:

EV3 = FV (1 + r/f)-n1 + FV (1 + r/f)-n2

We use the variables n1 and n2 to designate the number of periods


that were used to discount the two debts.

EV3 = 80,000(1 + 0.05/2)-2 + 50,000(1 + 0.05/2)-4


EV3 = 76,145.15 + 45,297.53
EV3 = 121,442.68

Example 6: JC borrowed P20,000 due at the end of Year 2. JC incurs another loan
from atom of P50,000 due at the end of Year 5. If the two agrees that
JC will pay his obligations in a single payment at the end of Year 4,
what equivalent amount is expected by Atom assuming that money
is worth 10% (settlement rate) compounded quarterly? Comparison
date = Payment date.

Solution: We first analyze the problem using a timeline:

66
Remember that P20,000 and P50,000 are maturity values of a certain
loans. However, P20,000 at Year 2 becomes present value with
reference to Year 2 and it is to be compounded/accumulated to
the 4th year which is the known date. There are cases when you are
asked to compute first for the maturity value of the debts before you
could discount or accumulate them on comparison date at a given
settlement rate. This was illustrated in Example 4.

In our problem, we compute the equivalent value at Year 4 by:

EV4 = FV(1 + r/f)-n + PV(1 + r/f)n


EV4 = 50,000(1 + 0.10/4)-4 + 20,000(1 + 0.10/4)8

We do not designate the variable n as n1 and n2 anymore because it


is obvious that we use both accumulation and discounting. Rather,
we designate the two by n and –n.

We discount P50,000 for 4 periods (1 year, quarterly) and we


accumulate P20,000 for 8 periods (2 years, quarterly);

EV4 = 45,297.53 + 24,368.06


EV4 = 69,665.59

Example 7: The following set of obligations is due on different dates:

Debt 1 MV = P30,000 ; with P invested at 8% compounded


annually after 3 years
Debt 2 MV = P56,000; with P invested at 9% compounded
semi-annually after 6 years
Debt 3 MV = P88,000; with P invested at 10% compounded
quarterly after 5 years

If at Year 4, the parties intend to settle the debts at a settlement rate


of 10% compounded semi-annually, determine the equivalent value
at Year 4.

Solution: EV4 = PV(1 + r/f)n + FV(1 + r/f)-n1 + FV(1 + r/f)-n2


EV4 = 30,000(1 + 0.10/2)2 + 56,000(1 + 0.10/2)-4 + 88,000(1 + 0.10/2)-2

With reference to Year 4, Debt 1 is to be accumulated as it is located


before the comparison date. Hence,

EV4 = 33,075 + 46,071.34 + 79,818.59


EV4 = 158,964.93

67
Example 7: Ms. Call has the following payables to Mr. Bean.

Maturity Value Due after
P30,000 5 years
P25,000 3 years
P145,000 9 years

If Ms. Call decides to settle the debts at a settlement rate of 10%


compounded semi-annually, what series of equal payments must be
made at Year 6 and 7 respectively? Comparison Date = Year 7.

Solution: Let us assume x as each payment and we use Year 7 as comparison


date. Therefore, there is a need to accumulate 30,000 and 25,000 and
to discount 145,000.

Hence, bringing the values to the comparison date, we have:

EV7 = 30,000(1 + 0.10/2)4 +25,000(1 + 0.10/2)8 +145,000(1 + 0.10/2)-4


EV7 = 36,465.19 + 36,936.39 + 119,291.86

This means that the value of the old debts at Year 7 is:

EV7 = 192,693.44

Since the value of each payment is x, we can deduce that since at Year 6, the
payment is x, the second payment then becomes x[(1+0.10)2] or x(1.1025)
because it is 2 periods close to Year 7, the comparison date. Hence, equating
the values of the old and new obligations:

x + 1.1025x = 192,693.44
2.1025x = 192,693.44
x = 91,649.67

This means that Ms. Call will pay Mr. Bean P91,649.67 at Year 6 and another
P91,649.67 at Year 7.

68
REVIEW QUESTIONS

1. Explain the concept of present value of an investment.

2. How does the present value concept apply to business practice?

3. State the formula to compute for the present value of an investment that involves
single receipt or payment (lump sum).

4. How to discount investments that involve fractional discounting periods.

5. How does present value concept apply to equations of values of a set of obligations.

69
PROBLEM 1. Present Value and Discount

Complete the table below.

Compound Annual Term Interest Present Compound


Amount Rate of Compounded Value Interest
Loan
1. P80,700 15% 4 years Bi-monthly
2. P25,900 8% 10 Quarterly
years
3. P56,360 12% 6 years Annually
4. P23,800 10% 5 years Semi-annually
5. P14,890 13% 3 years Quarterly

PROBLEM 2. Present Value, Discount, Time and Interest Rate.

Fill in the missing items in the table.

Compound Annual Term Interest Present Compound


Amount Rate of Compounded Value Interest
Loan
1. P65,900 4 years Monthly 50,000 15,900
2. P29,750 8% Quarterly 20,000 9,750
3. P32,300 9% 6 years Annually
4. P45,800 10% Semi-annually 30,000 15,800
5. P56,900 3 years Quarterly 30,000

70
PROBLEM 3. Present Value, Discount, Time and Interest Rate.

1. Jubail invested in a fund where he has received a total of P45,800 after 2 years
inclusive of principal and interest. If the bank pays 8% compounded semi-
annually, how much was invested by him?

2. Rocco invested in a bank and expects an amount of P80,500 compound amount


in the future. If the term of the investment is 2 years and interest is compounded
quarterly, at what interest rate would the return be possible?

3. Sunday borrowed an amount with maturity value of P56,980 from Tuesday. If


Tuesday wants a return of 8% compounded semi-annually for 3 years, what is
Sunday’s amount of loan from Tuesday.

4. Dora borrowed from Diego an amount of P30,000 with a an interest rate of


8% compounded semi-annually. Diego wants that the principal together with
an interest of P9,780 be collected from Dora at the end of the term. At what
investment term could these premises be possible?

5. Discount P54,500 for 10 years at 4% compounded annually. What is the amount


of discount?

71
6. Find the present value of P90,867 due at the end of 9 years if money is worth 8%
compounded semi-annually.

7. Mrs. Heral just gave birth to a son. Mr. Heral plans to give his son an amount of
P1,000,000 at his 21st birthday. If a bank pays 6% interest every half-year, what
amount of money should Mr. Heral deposit now for him to accumulate the said
gift amount?

8. Acel Adelyne, at age 20, plans to deposit an amount which will yield her a total
amount (principal plus interest) of P200,000 on her 30th birthday. How much
must be invested now if money is worth 6% compounded quarterly?

9. When is P56,000 due if its present value is P20,000 when money’s value is 5%
compounded annually?

10. How long would it take for an investment of P40,000 to double is invested at a
rate of 8% compounded semi-annually?

72
11. What is the present value of an amount of P89,000 due after 3 years if it was
invested at a rate of 9% compounded semi-annually? Would your answer
change if it was invested compounded quarterly? By how much is the increase/
decrease?

12. Your company offered you monetization plans on your retirement benefit
claims. You were offered P5,000,000 now or P10,000,000 after 5 years. If money
is worth 15% compounded quarterly,

a.) Which plan is more favorable?

b.) By how much advantage?

c. Would your answer change if interest rate becomes 10%? By how much is
the increase/decrease?

d. Analyze and explain how interest rate affects the value of investments.

PROBLEM 4. Equivalent Values

1. Samson has the following schedule of loans from Delilah:

Due after Maturity Value, in PhP Terms


2 years 20,000 5% compounded annually
4 years 30,000 8% compounded annually
6 years 15,000 10% simple interest
8 years 25,000 8% compounded semi-annually
10 years 30,000 9% compounded quarterly

If at Year 6 Samson wants to settle this set of obligations into a single payment,
what equivalent value would discharge this set of debts if money is worth 9%

73
compounded quarterly? Use a timeline to illustrate the process to be used then
compute the single equivalent value using the formula.

2. In the same problem with item # 1, except that the settlement date is at Year
7, and the comparison date is at Year 8, determine the single equivalent value
that would discharge the debts of Samson.

3. What single payment will now settle the following obligations with a
settlement rate of 8%compounded semi-annually; P5,000 due after 5 years,
P6,000 due after 2 years and P9,000 due after 3 years?

4. An obligation which is worth P150,000 will be paid by installment. The


following are the schedule of payments:

Due at the end of 3 years P20,000;


Due at the end of 5 years P25,000;
Due at the end of 6 years P50,000.

74
In addition, a final payment is due at the end of 8 years.

Find the final payment if money is worth 8% compounded annually and the
comparison date is at Year 4.

5. Sarah owes Charice the following set of debts:

Maturity Value Due after


2,000 3 years
9,000 at 5% compounded annually 3 years
5,000 at 7% compounded semi-annually 6 years
5,000 at 8% simple interest 8 years

If money is worth 8% compounded semi-annually, what single payment at


the end of 7 years will equitably replace the obligations?

75
CHAPTER IV

SIMPLE ANNUITY
LEARNING OBJECTIVES

• Describe the concept of simple annuity, its’ nature and classifications.


• Understand how annuities are applied to business practice.
• Acquire basic knowledge on how to compute for the future and present value of
an ordinary annuity.
• Determine how to compute for the future and present value of an annuity due.
• Determine how to manipulate the simple annuity formulas.
Let us consider the following investment decisions:

Investment 1: You are planning to buy a life insurance. You were provided with
a list of available insurance plans. How much should you invest
every month? Every quarter?

Investment 2: You retired from work. You consulted your life insurance company
regarding your retirement benefits which accumulated to P5,000,000.
The company offered you two monetization plans. You have the
option to receive P5,000,000 now or to receive P350,000 per quarter
for 5 years. Which would you choose considering the desirability of
both options?

What we have discussed in the preceding chapters involve lump-sum or single


receipt or payment. However, in business practice there are transactions which
will require series of equal sum of payments or receipts at intervals for a certain
time period. These series of equal payments or receipts is generally referred to as
annuities

Annuities refer to series of payments or receipts usually made at equal amounts


and at equal intervals for a certain time period. It is called simple annuity; but,
when payment interval does not coincide with interest conversion, it is called
general annuity. In business practice, it is usually used in the case of insurance
premiums, lottery winnings, insurance benefits and the like. There are various
types of annuities. In this chapter, we will only be dealing with ordinary annuity,
annuity due and deferred annuity.

An annuity is classified as an Ordinary Annuity if the payments or receipts are


made at the end of each payment interval. On the other hand, an annuity is an
Annuity Due if payment or receipts are made at the beginning of each payment
interval while in the case of Deferred Annuity, payment or receipts are made at the
end of each payment interval however, the term of the loan is deferred or does not
begin until a certain agreed period.

Annuities are also classified by term as in the case of Annuity Certain, when
the term is fixed; Contingent Annuity, when it depends on the happening of an
uncertain event; and Perpetuity in which the term never ends.

The time elapsed between the payments/receipts is called payment interval while
the time between the first and last payment is called the term of the annuity.

ORDINARY ANNUITY

An annuity is classified as an Ordinary Annuity if the periodic payments/receipts

78 78
are made at the end of each payment interval. If not
specified, payments are assumed to be made at the end of
each payment interval.

FUTURE VALUE OF AN ORDINARY ANNUITY

The Future Value of an ordinary annuity can be computed


similarly as in the case of single payment. However, it involves the addition of
the future value of each payment/receipt to arrive at the total future value of the
annuity.

To illustrate, we use our first example:

Example 1: Jaar deposits P2,000 every end of the year for 5 years. If money is
worth 10% compounded annually, determine the future value of the
annuity.

Solution: The future value of each periodic payment could be separately


computed (as in the case of single payment) and the resulting
amounts are totaled to obtain the total future value.

Period, n Individual Computation (Single Future


Payment) Value
1 FV = 2,000(1 + .10/1)0* 2,000.00
2 FV = 2,000(1 + .10/1)1 2,200.00
3 FV = 2,000(1 + .10/1)2 2,420.00
4 FV = 2,000(1 + .10/1)3 2,662.00
5 FV = 2,000(1 + .10/1)4 2,928.20
Total 12,210.20

*n = 0 because deposit is made at the end of the period, hence, there is no


time for the investment to gain interest.

This computation works well when payment interval and the term of the annuity
involves only a short period of time. When payment interval becomes 30, 50 or
even 100, this method becomes impractical; hence mathematicians derived the
following formula in determining the future value of an ordinary annuity.

79
Formula 1: Future Value of an Ordinary Annuity

FVOA = Pmt [ (1+r/f)n – 1]
r
where:

FVOA future value of an Ordinary Annuity

Pmt periodic payments/receipts

r interest rate; expressed as periodic interest

n total number of payment interval
f conversion frequency per year

Hence, in our example;

FVOA = Pmt [(1+r/f)n – 1]


r

FVOA = 2,000 [(1+0.10/1)5 – 1]


0.10

FVOA = 12,210.20

Example 2: Pettizou wants to buy a lot after 5 years. If she invests P10,000 at
the end of every quarter, and if money is worth 8% compounded
quarterly, determine the future value of the investment.

Solution: Using the formula, given that Pmt = 10,000, t = 5 years, f = 4, r = 8%


and n = 20; the future value is determined as follows:

FVOA = Pmt [(1+r/f)n – 1]


r

FVOA = 10,000 [(1+0.08/4)20– 1]


0.08/4

FVOA = 242,973.70

Example 3: Daryl wants P5,000,000 at the end of 3 years. If money is worth 10%
compounded quarterly, what quarterly deposits are required for
him to have the desired amount after 3 years?

80
Solution: The problem simply calls for the computation of periodic payments.
Hence, given that FVOA = 5,000,000, f = 4, t = 3 years, n = 12, and r =
10%; periodic deposit is computed as:

FVOA = Pmt [(1+r/f)n – 1]


r

5,000,000 = Pmt [(1+0.10/4)12– 1]


0.10/4

5,000,000 = Pmt [(1.025)12– 1]


0.025

By cross multiplication, we have;

125,000 = Pmt [(1.025)12– 1]


125,000 = Pmt (0.34488824)

Divide both sides by 0.34488824, we have;

Pmt = 362,435.63

Note: We use the variable Pmt to designate either payments or receipts. Hence,
an annuity investment problem must be carefully analyzed for a student to
identify what is asked in the problem.

PRESENT VALUE OF AN ORDINARY ANNUITY

We are now done with the computation of future value of an ordinary annuity. The
computation of the Present Value of an ordinary annuity presents no complicated
problem since a formula is available. To find the present value of an ordinary
annuity, we use the following formula:

81
Formula 1: Present Value of an Ordinary Annuity


PVOA = Pmt [ 1 - (1+r/f)-n]
r

where:

PVOA present value of an Ordinary Annuity

Pmt periodic payments/receipts

r interest rate; expressed as periodic interest

n total number of payment interval
f conversion frequency per year

To illustrate the use of the formula, let’s try an example:

Example 4: Cameron deposits P6,000 every month for 5 years in a fund which
pays an interest of 9% compounded monthly. Determine the present
value of the deposits.

Solution: Given that Pmt = 6,000, t = 5 years, f = 12, n = 60, and r = 9%; the
present value of the periodic deposits is computed as:

PVOA = Pmt [ 1 - (1 + r/f-)-n]


r

PVOA = 6,000 [1 - (1 + 0.09/12)-60]
0.09/12

PVOA =289,040.24

Example 5: Justine accumulated 2,000,000 for retirement. If he invests this to


a fund which pays 10% interest compounded semi-annually, how
much would he withdraw for his money to last for 7 years?

Solution: The problem simply asks for periodic withdrawals to be made to


bring the balance of the investment to zero after 7 years, that is for
14(7 x 2) periods. Since the amount is stated in present value, periodic
withdrawal is computed on the basis of equating the present value

82
with the present value factor. Hence, given that PVOA = 2,000,0000, t
= 7 years, f = 2, n = 14, and r = 10%, Pmt is computed as:

PVOA = Pmt [1 - (1+r/f-)-n]


r

2,000,000 = Pmt [1 - (1 + 0.10/2)-14]
0.10/2

By cross multiplication, we have;

100,000 = Pmt [1 - (1 + 0.10/2)-14]


100,000 = Pmt (0.494932047)

Dividing both sides by 0.494932047, we have;

Pmt = 202,047.94

Note: It is extremely important for the learner to identify the nature of an amount
given whether it is on its present or future value. The amount must be
consistent with its factors in the equation as in the case of finding other
variables other than future and present value like r and n. The analysis and
explanation of why the present value formula is to be used in solving the
preceding problem is discussed in the succeeding chapter.

UNKNOWN RATE OF INTEREST OF AN ORDINARY ANNUITY

There are times that a problem would require us to compute for other unknown
variables such as the interest rate and term of an ordinary annuity. The computation
of which presents no complicated problem because like single payments, same
computation process is used.

When interest rate is unknown in an ordinary annuity investment problem, the


Direct Method which involves extracting the roots of both sides of the equation
could not be used because the formula has two r variables. Hence, in this case,
linear interpolation is used.

Example 6: An annuity investment of P1,000 payable every quarter-end will


amount to P45,894.90 after 5 years. At what rate compounded
quarterly is this yield possible?

Solution: We get first the Table Factor by dividing the Future Value of the
Ordinary Annuity by the Annuity Payment. That is:

83
[(1+r/f)n – 1] = FVOA /Pmt
r

[(1+r/f)n – 1] = 45,894.90 /1,000


r

[(1+r/f)n – 1] = 45.8949
r

We scan across Period 20 in Table III and that 45.8949 can be seen
between 8% and 9% whose factors are 45.762 and 51.160 respectively.
We present the factors in tabular form:

Interest Table factor or


Rate per [(1+r/f)n – 1]/r
period
9% 51.160 (1)
r 45.8949 (2)
8% 45.762 (3)

(2) – (3) r – 8% = 0.1329 (4)


(1) – (3) 1% 5.398 (5)

By cross multiplication;

(1% x 0.1329) = (r – 8% )(5.398)


0.001329 = 5.398 r – 0.43112

By transposition;

5.398 r = 0.001329 + 0.43112


5.398 r = 0.432449

Dividing both sides by 5.398, the value of r is:



r = 0.080112819 x 100
r = 8.01%

When a problem calls for finding the interest rate and the present value of an
ordinary annuity is given, same procedures are to be followed in interpolation of
values except that the factors to be used will be the factors from Table IV.

UNKNOWN TERM OF AN ORDINARY ANNUITY

Finding unknown time or term of an ordinary annuity presents no problem


because the methods available in single payment computation work the same with

84
the ordinary annuity formula. Let us try first the use of logarithm.

Example 7: An annuity investment of P2,000 every end of half-year will


accumulate to P86,590 at the end of a certain time. With an interest
of 8% convertible quarterly, what must be the term?
Solution: Given that FVOA = 86,590, Pmt = 2,000, r = 8% and f = 4; we compute
the term by:

FVOA = Pmt [(1+r/f)n – 1]


r

86,590 = 2,000 [(1+0.08/4)n– 1]


0.08/4

By cross multiplication, we have:

1,731.80 = 2,000 [(1+0.08/4)n– 1]



We divide both sides by 2,000, we have:

0.8659 = (1+0.08/4)n – 1

By transposition:


0.8659 + 1 = (1.02)n

1. 8659 = (1.02)n

log 1.8659 = log (1.02)n


log 1.8659 = n log 1.02 log ax = xlog a

log 1.8659 = n log 1.02


log 1.02 log 1.02

n = log 1.8659
log 1.02
n = 31.50

Since the compounding frequency is quarterly, we divided the


resulting value of n by 4 for us to arrive at the term. Thus;

t = 7.88 years

ANNUITY DUE

An annuity due is a type of annuity in which the periodic payments are made at the
beginning of each payment interval. In this type of annuity, the term commences

85
on the first payment and ends one payment interval after the last payment period.

FUTURE VALUE OF AN ANNUITY DUE

The computation of the future value of an annuity due is similar to the computation
of future value of an ordinary annuity. The formula is presented below:

Formula: Future Value of an Annuity Due



FVAD = Pmt [ (1 + r/f)n – 1] x (1 + r)
r
where:

FVAD future value of an Annuity Due

Pmt periodic payments/receipts

r interest rate; expressed as periodic interest

n total number of payment interval

f conversion frequency per year

The formula adds (1 + r) because of the interest earned for one additional period
since periodic payments are made at the beginning of each payment interval.

Example 8: What is the future value of an annuity of P3,000 every beginning


of the month for 3 years if invested at a rate of 8% compounded
monthly?

Solution: Given that Pmt = 3,000, r = 8%, t = 3 and f = 12;

FVAD = Pmt [(1 + r/f)n – 1] x (1 + r)


r

FVAD = 3,000[(1 + 0.08/12)36 – 1] x (1 + 0.08/12)


0.08/12

FVAD = 122,417.38

PRESENT VALUE OF AN ANNUITY DUE

The computation of present value of an annuity due is somewhat similar with

86
that of ordinary annuity except for the interest factor that is added in the formula.
To compute for the present value of an ordinary annuity, we use the following
formula:

Formula: Future Value of an Annuity Due




PVAD = Pmt [1 - (1 + r/f)-n] x (1 + r)
r

where:

PVAD future value of an Annuity Due

Pmt periodic payments/receipts

r interest rate; expressed as periodic interest

n total number of payment interval

f conversion frequency per year

To illustrate the use of the formula, let us try an example:

Example 9: Determine the present value of an annuity due with periodic


payments of P2,500 every beginning of a quarter for 3 years at an
interest of 9% compounded quarterly.

Solution: Given that Pmt = 2,500, r = 9%, t = 3 and f = 4;



PVAD = Pmt [1 - (1 + r/f)-n] x (1 + r)
r

PVAD = 2,500 [1 - (1 + 0.09/4)-12] x (1 + 0.09/4)


0.09/4

PVAD = 26,622.78

UNKNOWN RATE OF INTEREST OF AN ANNUITY DUE

Example 10: At what nominal rate compounded annually will an annuity in


advance of P5,000 payable every beginning of the year for 5 years

87
amount to P90,000.
Solution: Given that FVAD = 90,000, Pmt = 5,000, t =11 (10+1) and f = 1;

FVAD = Pmt x Table Factor - Pmt

FVAD + Pmt = Pmt x Table Factor

Table Factor = FVAD + Pmt


Pmt

Table Factor = 19

Scanning across the values on Table V, we can locate 19 and therefore


approximate that it is between 8% and 9%

UNKNOWN NUMBER OF PERIODS OF AN ANNUITY DUE

Example 11: Jonnel acquired a unit of the latest mountain bike. It costs P150,000.
He agreed to pay in installment amounting to P15,500 a year. If
interest is 10% compounded annually, what was the term agreed?

Solution: Given that PVAD = 150,000, Pmt = 15,500 and i= 10%;

PVAD = Pmt x Table Factor + Pmt



FVAD - Pmt = Pmt x Table Factor

Table Factor = PVAD - Pmt


Pmt

Table Factor =8.677

Scanning across the values on Table VI, we can locate 8.677 between
16 and 17.

Rule: Since 8.677 lies between two certain interest periods, it implies
that the creditor wants a return of greater than 16% but less than 17%
or simply, the value of (n-1). We do not need to interpolate, but only
to conclude that we use the greater period for the creditor to fully
recover his investment. Hence, if n-1 = 17, n therefore is 18.

Additional Topic: DEFERRED ANNUITY

The preceding discussions on Ordinary Annuity and Annuity Due presents a


simple approach to compute for their present and future values and that the main

88
difference lies on when the periodic payments are made. In business practice
however, there are some annuity investments that its term starts on a future date.
For example, an annuity investment is bought now (assuming beginning of the
year) which is to be funded monthly for one (1) year; but the first periodic payment
is to be made 2 months later. In this case, the annuity is said to be deferred for two
months and the term starts from the third month extending up to the first two
months of next year. The first two months in the example is referred to as period
of deferment.

FUTURE VALUE OF A DEFERRED ANNUITY USING THE TABLE

Like in our previous discussions, the computation of the future value of a deferred
annuity presents no complex calculations. The future value of a deferred annuity
is the final amount of the annuity at the end of its term. The future value of a
deferred annuity is the same as the future value of an ordinary annuity. Hence, the
deferment period is disregarded.

Illustration: Find the final amount of a P2,000 annuity due every end of the
month for 9 payments with the first payment made at the end of
the third month. Interest rate is 12% compounded monthly.

Solution: Given that Pmt = P2,000; t = 0.75 years; n = 9; r = 12%; f = 12, we


have:

FVDA = Pmt x Table Factor

Using the factor from Table III;

FVDA = 2,000 x 9.369


FVDA = 18,738

PRESENT VALUE OF A DEFERRED ANNUITY USING THE TABLE

Illustration: An annuity payment of P2,000 to be made every end of the quarter


for 8 quarters. If interest is 2% compounded quarterly, find the present
value of the annuity investment if the first payment is made at the third period.

The present value of a deferred annuity can be computed by following the steps:

1. Determine the deferment period. Add the deferment period with the term.
In our example, n = 8, and deferment period is 2; hence, our new n = 10.
2. Using Table IV, find the corresponding factor for the new n. In our example,
we have: Table Factor = 8.530

89
3. Determine the table factor of the deferment period as if used separately;
that is, Table Factor = 1.913
4. Use the formula PVDA = Pmt x Difference in Table Factors:
PVDA = 2,000 x (8.530 – 1.913)
PVDA = 13,234

REVIEW QUESTIONS

1. Explain the concept of simple annuity?

2. What are the different types of simple annuities? How do they differ?

3. What are the usual applications of simple annuities in busines practice?

4. State the formula to compute for the present value future value of ordinary
annuity? What about the annuity due?

5. How do we compute for the present and future value of a deferred annuity?

6. How do you compute for unknown variables in the present and future value
formula of ordinary annuity and annuity due?

90
PROBLEM 1. Future Value of Ordinary Annuity

1. A 5-year ordinary annuity has a present value of P1,000. If the interest rate
is 8 percent, how much is the amount of each annuity?

2. What amount will accumulate if we deposit P5,000 at the end of each year
for the next 5 years? Assume an interest of 6% compounded annually.
Complete the table.

Year 1 2 3 4 5
Begin 0 5,000 10,300
Interest 0 955.08
Deposit 5,000
End 28,185.46

3. What is the accumulated value of a P2,500 payment to be made at the


end of each of the next three years if the prevailing rate of interest is 9%
compounded annually?

4. At the end of the next 5 years, Mr. Y will deposit P3,000 every month.
Determine the future value of his annuity investment if the interest rate
prevailing is 3% compounded monthly.

5. Yen deposits P2,000 every end of the quarter. The interest rate prevailing
for the same type of investment is 4%. Determine the future amount if the
term calls for 36 months.

6. Determine the future value of an annuity of P5,000 made every month-end


if interest is worth 5% compounded monthly for 4 years.

91
7. Angeline invests in a fund that requires quarterly deposits of P3,000. If the
fund pays interest of 5% quarterly, what is the future value assuming it will
be held for 5 years.

8. Phylie makes deposits of P2,500 every month. The fund pays interest of
2%monthly. If the investment is held for 5 quarters, compute for the future
value.

9. Red deposits every half-year P10,000 in an annuity account. The account


will pay Red 8% interest compounded semi-annually. If the term is 5 years,
compute for the future value.

10. Miles invested P2,000 each month in a fund that pays 3% interest
compounded monthly. If she managed to invest for 3 semi-annuals, how
much will she accumulate at the end of the investment’s term?

PROBLEM 2. Present Value of an Ordinary Annuity-Manipulating the Formula

1. Mr. Y buys a real estate and paid cash amounting to P200,000 and the
balance to be paid at annual installment of P50,000 for 5 years. If the interest
rate is 8% compounded annually, determine the cash price equivalent of
the real estate.

92
2. Suppose you win a lottery that entitles you to receive $500 per month for
the next 20 years. If money is worth 6% compounded monthly, what is the
present value of this annuity?

3. Which would you prefer assuming an interest rate of 9% compounded


annually.
a. To receive P500,000 now or
b. To receive P30,000 every year for 20 years?

4. Which would you prefer assuming an interest of 9% compounded annually.


a. To receive P30,000 every year for 2 years or
b. To receive P55,000 now

5. Mr. D estimates that he will receive P9,000 pension every month for the
next 10 years. If the market rate is 4% compounded monthly, determine the
present value of the pension he will receive.

6. Dee acquires a house. The agreed payment terms provide for a down
payment of P200,000 and the balance to be paid at 4 quarterly payments of
P30,000. If the interest rate agreed is 4% compounded quarterly, determine
the present value of the annual installments.

93
7. If X expects a present value of P40,000 of an annuity investment to be held
for 4 years, how much will he have to deposit monthly to equate the said
amount now earning 5% interest compounded monthly for 4 years?

8. Manny pays P2,000 in a fund for every month-end earning interest of 5%


compounded monthly. If the term is 4 years, determine the investment’s
present value.

9. Find the quarterly payment of a 5% compounded quarterly investment if its


present value amounts to P35,880. The investment will be held for 5 years.

10. Determine the present value of P3,000 invested at 5% compounded monthly


if the invested will be held for 5 quarters.

PROBLEM 3. Unknown Interest Rate of an Ordinary Annuity

1. An annuity investment of P20,000 payable every month-end will amount


to P860,000 after 5 years. At what rate compounded quarterly is this yield
possible?

94
2. What rate of interest compounded semi-annually will an annuity of P6,000
payable every end of six months amount to P87,600.

3. The present value of an ordinary annuity is P89,670. It’s periodic payment is


P5,000 every quarter-end for 5 years. At what rate is this scenario possible?

4. An annuity investment every end of the quarter requires payment of P8,900.


The value now of these payments is P90,880. If the investment is held for 3
years, what is the interest rate compounded quarterly?

5. Andrea bought a furniture which costs P90,000. She made a down payment
of P10,000 and the balance is payable in installment amounting to P11,892
every quarter-end for 2 years. Compute for the interest rate.

PROBLEM 4. Unknown Term of an Ordinary Annuity

1. Assume that you borrow P700 from a friend and intend to repay the amount
in equal installments of P100 per year over a period of years. The payments
will be made at the end of each year beginning one year from now. Your
friend wishes to be reimbursed for the time value of money at a 7% annual
rate. How many years would it take before you repaid the loan?

2. How long will it take for a semi-annual deposit of P500 to accumulate to


P67,897 if money is worth 8% compounded semi-annually.

95
3. P23,908 is the present value of a P1,245 deposit every end of the month. It
is compensated by 12% compounded monthly. What is the time required to
make this yield possible?

4. How many years will it take to accumulate P98,750 if P2,590 is deposited


at the end of each quarter if interest rate of 10% converted quarterly? How
about if deposit and compounding is made annually?

5. After retirement, Gary place his retirement benefits amounting to P3,000,000


in a fund that pays 10% interest compounded monthly? How long will it
take for him to exhaust the fund if he withdraws P5,000 every month?

PROBLEM 5. Unknown Term of an Ordinary Annuity

Compute for the time required

Payment Periodic rate Future Value Present Value


P3,000 10%, f = 1 P89,000
P4,200 2%, f = 4 P94,000
P3,200 5%, f = 2 P230,000
P2,100 4%, f = 2 P77,000
P3,020 6%, f = 2 P96,000

96
PROBLEM 6. Future Value of an Annuity Due

1. Cherry deposits P2,000 every beginning of month. Determine the Future


Value of the said annuity deposits if it earns 9% compounded monthly for
5 years.

2. A fund was established that requires P3,000 quarterly deposits. Deposits


are made every beginning of the period. Calculate for the Accumulated
Value of the deposits after 10 deposits.

3. An annuity deposit of P2,690 is made every beginning of the quarter for 5


years. Determine the final amount of the investment if it earns 10% interest
compounded quarterly.

4. Carlo receives every beginning of the month an amount of P2,000. He


deposit it in a fund that pays 10% interest compounded monthly. Determine
the future value after 10 years.

5. An annuity policy provides an annuity payment P5,000 every half-year.


Payment is made every beginning of the period. If it pays 12% semi-
annually, determine the investment’s future value.

97
PROBLEM 7. Future Value of Annuity Due

Complete the Table

Payment/Receipt Periodic Rate Term Future Value


P9,000 4%, f = 2 3 years
P2,000 6%, f = 2 4 quarters
P2,300 3%, f = 4 2 semi-annual
P1,200 10%, f = 1 10 years
P4,800 2%, f = 4 6 semi-annual

PROBLEM 8. Present Value of Annuity Due

Complete the Table: Cash flow is made/received at the beginning of period.

Payment/Receipt Periodic Rate Term Present Value


P3,000 4%, f = 2 3 years
P2,900 6%, f = 2 4 years
P1,800 3%, f = 4 5 years
P2,200 10%, f = 1 2 years
P8,000 2%, f = 4 8 years

PROBLEM 9. Present Value of Annuity Due

1. A television set that costs P90,000. The buyer paid P10,000 and the balance
to be paid in 9 equal installments for 6 quarters (every beginning of the
period). If interest paid is worth 10% compounded quarterly, determine
the present value of the installment payments. Compute for the cash price
equivalent of the set.

98
2. Find the present value of a series of payments made every beginning of
the quarter. The annuity payments amounts to P5,000. If interest is 9%
compounded quarterly, determine the present value of the cash flow if
made for 5 years.

3. A man will receive a series of equal receipts every beginning of the month
which amounts to P2,000. If money is worth 12% compounded monthly,
determine the present value if the receipts is to be received for 6 years.

4. A retired employee will receive a monthly pension of P2,300. The pension


is to be received every beginning of the month. Determine the present value
of the annuity receipts if it is to be received for a fixed term of 10 years.

5. The present value of an annuity payment is P48.908. If interest is 12%


compounded monthly, find the periodic payments made every beginning
of month every 8 years.

PROBLEM 10. Interest Rate of Annuity Due

1. At what nominal rate compounded semi-annually will an annuity in


advance of P3,000 payable every beginning of every 6 months for 5 years
amount to P80,000.

2. Mr. King deposits P2,500 in a fund made every beginning of the quarter.
The deposits will accumulate to P120,000 if made continuously for 10 years.
What must be the interest rate?

99
3. The present value of annuity payments of P1,000 every beginning of the
month is P89,880 if made for 7 years. What interest rate is used?

4. The future value of a given annuity due is P70,540. Deposits of P2,900 are
made every month for 6 years. Determine the rate used.

5. Lovi deposits in a fund which requires P1,980.22 deposits every month.


Deposits are made at the beginning of the month. Determine the rate used
if the annuity payments accumulates to P165,900 at the end of 5 years.

Payment Periodic Term Compounding/payment Future Present


rate (years) frequency per year Value Value
P4,000 4 4 P90,870
P1,200 6 2 P54,000
P9,200 10 1 P230,000
P2,100 5 4 P88,000
P1,020 1.5 2 P56,000

PROBLEM 11. Term of Annuity Due

Compute for the time required (payment in advance). Answer all missing data.

Payment Annual rate Future Value Present Value


P3,000 10%, f = 1 P65,000
P4,200 12%, f = 4 P94,000
P3,200 15%, f = 2 P230,000
P2,100 14%, f = 2 P77,000
P3,020 13%, f = 2 P96,000

1. P23,908 is the future value of a P1,245 deposit every beginning of quarter.


It is compensated by 12% compounded monthly. What is the time required
to make this yield possible?

100
2. How many years will it take to accumulate P98,750 if P2,590 is deposited
at the beginning of each quarter if interest rate of 13% converted quarterly?
How about if deposit and compounding is made annually?

3. Buddy’s retirement plan allows him to receive P3,000 each every beginning
of the month. If the PV of the annuity due receipts is P1,890,000 and 10%
interest is compounded monthly? How long will it take for him to exhaust
the fund?

4. Assume that you borrow P2,000 from a friend and intend to repay the
amount in equal installments of P100 per year over a period of years. The
payments will be made at the beginning of each year beginning one from
now. Your friend wishes to be reimbursed for the time value of money at a
7% annual rate. How many years would it take before you repaid the loan?

5. How long will it take a semi-annually deposit of P500 to accumulated to


P157,200 if money is worth 10% compounded semi-annually. Payment is
made in advance.

101
PROBLEM 12. Deferred Annuity-Future and Present Value

Hint: Table factors can be obtained using a calculator.

102

CHAPTER V

AMORTIZATION AND
SINKING FUND
LEARNING OBJECTIVES

• Understand the concept of discharging present financial obligations through


amortization.
• Describe the concept of discharging future financial obligations through
establishment of sinking fund.
• Acquire basic knowledge on how to construct sinking fund and amortization
schedules.
AMORTIZATION

Suppose you have an interest-bearing debt. To discharge the debt, the terms of the
loan provide that it is to be repaid in equal installments at regular intervals. This
discharging of debt by sequence of equal payments at regular intervals is called
amortization.

In essence, paying off a debt using amortization forms an annuity payment. The
payments include a portion of the principal and interest being repaid. The present
value then of these payments is equal to the original loan.

To compute the amount of amortization payment, we use the following formula:

Formula 1: Amortization Payment


AP = PV x r
1 - (1 + r/f)-n

where:

AP amortization payment

PV present value

r interest rate; expressed as periodic interest

n total number of payment interval

f conversion frequency per year

We observe that the formula for the amortization payment is very much similar
with the formula for finding the periodic payments for ordinary annuity when
present value is given. The formula is similar since amortization payment is a form
of annuity. This formula is applicable when amortization payments are made at the
end of every period. Hence, if amortization payments are made at the beginning of
each period, we derive the formula for payment, by using the formula for annuity
due.

To illustrate the use of the formula, let us try an example.

104 104
Example 1: Herson bought a refrigerator costing P50,000. He paid a down
payment of P10,000 and agreed to pay the remaining amount by
making quarter-end payments for 2 years. If the interest for the
said balance is 10% compounded quarterly, determine the size of
quarterly payments.
Solution: Given that PV = 40,000, that is 50,000-10,000; t = 2, f = 4 and r = 10%,
AP is computed as follows:

r
AP = PV x 1 - (1 + r/f)-n

10%/4
AP = 40,000 x 1 - (1 + 10%/4)-8

0.025
AP = 40,000 x 1 - (1.025)-8

AP = 5,578.69

OUTSTANDING PRINCIPAL

Each amortization payment in the amortization method of discharging debt pays


off a portion of the original principal and interest. With this in mind, there are
times that a debtor would want to know the principal outstanding after each
amortization payment.

The outstanding principal may depend on the agreement made by the parties. First,
is when the parties agree at a certain fixed amount as periodic payments, except
for the last payment that is adjusted by amount to reflect the remaining balance to
be paid. Second is that the periodic payments are mathematically calculated using
the formula given above.

The amortization of debt can be presented using a table. Consider the example
below:

Example 2: Periodic Payments Using the Formula- A loan of P20,000 is to be


amortized at the end of each semi-annual for 2 years. If the interest
rate is 10% compounded semi-annually, determine the size of each
periodic payment. Construct an Amortization Schedule.

Solution: Given that PV = 20,000, t = 2, f = 2 and r = 10%

We determine AP as follows:

105
r
AP = PV X 1 - (1 + r/f)-n

10%/2
AP = 20,000 x 1 - (1 + 10%/2)-4

0.05
AP = 20,000 x 1 - (1.05)-4

AP = 5,640.24

Payment Outstanding Interest Amortization Principal Repaid
Period, Principal at due at the Payment at the end of
n the beginning end of each period
of each period
period
(a) (b) (c) (d) (e)
(b-e) (b x .05) (d-c)
1 20,000 1,000 5,640.24 4,640.24
2 15,359.76 767.99 5640.24 4,872.25
3 10,487.51 524.38 5,640.24 5,115.86
4 5,371.65 268.58 5,640.23* 5,371.65
Total 2,560.95 22,560.95 20,000

*Rounding difference

Example 3: Periodic Payments with Agreed Equal Periodic Payments-Assume the


previous example, except that the agreed amortization payment is
P5,500. Complete the amortization schedule.

Solution: Since the amortization payment is given, there is no need anymore


to compute for it hence; we now construct an amortization schedule.

Payment Outstanding Interest Amortization Principal Repaid


Period, Principal at due at the Payment at the end of
n the beginning end of each period
of each period
period
(a) (b) (c) (d) (e)
1 20,000 1,000 5,500 4,500
2 15,500 775 5,500 4,725
3 10,775 538.75 5,500 4961.25
4 5,813.75 290.69 6,104.44 5,813.75
Total 2,604.44 22,604.44 20,000

106
SINKING FUND

When a borrower of funds obtain money from his creditor, he usually binds his
obligation by giving an instrument (such as notes) and a collateral or anything
that has value that could serve as security for the creditor in case of non-payment.
However, in some instances, especially when large amounts of long-term obligation
are concerned, creditors require their debtors to establish what we call a sinking
fund.

Sinking fund is a fund established to retire a long-term obligation in the future by


making periodic deposits at a time interval. The fund is invested to earn interest
that could serve as payment of the obligation at maturity. Take note that only the
principal amount is being paid out of the fund and not the interest.

Sinking fund may either be mandatory or voluntary. A mandatory sinking fund is


one which is required as per loan covenant between the contracting parties while a
voluntary sinking fund is established at the discretion of the borrowing company.

While amortization discharges a known amount of obligation at present, sinking


fund works otherwise. Sinking fund is established to discharge future amount,
hence, the formula to compute for the periodic payment is as follows:

Formula 1: Sinking Fund Payment



SFP = FV x r
(1 + r/f)n – 1

where:

SFP amortization payment

FV present value

r interest rate; expressed as periodic interest

n total number of payment interval

f conversion frequency per year

Let us try some examples:

Example 4: Mayumi Corporation will need P10,000,000 after 6 years to pay a loan
made for their plant expansion. The loan agreement provides that
the company will establish a sinking fund. What periodic payment

107
is needed every year at 9% compounded annually to meet the said
amount after 6 years? Construct a sinking fund schedule.

Solution: Given that FV = 10,000,000, t = 6 years, f = 4 and r = 9%, sinking fund


payment is computed as:
R
SFP = FV x (1 + r/f)n – 1

0.09/1
SFP = 10,000,000 x (1 + 0.09/1)6 – 1

0.09
SFP = 10,000,000 x (1.09)6 – 1

SFP = 1,329,197.83

Payment Sinking Interest in Periodic Accumulated Book Value


Period, n Fund Sinking Increase in Amount of Sinking of the Loan
Payment Fund Fund Fund
(a) (b) (c) (d) (e) (f)
(e x 0.09) (b + c) (d + e of previous
period)
0 0.00 0.00 0.00 0.00 10000000.00
1 1,329,197.83 0.00 1329197.83 1329197.83 8670802.17
2 1,329,197.83 119,627.80 1448825.63 2778023.46 7221976.54
3 1,329,197.83 250,022.11 1579219.94 4357243.41 5642756.59
4 1,329,197.83 392,151.91 1721349.74 6078593.14 3921406.86
5 1,329,197.83 547,073.38 1876271.21 7954864.36 2045135.64
6 1,329,197.86* 715,937.79 2045135.65 10000000.00 0.00
Total 7,975,187.00 2,024,813.00 10,000,000.00

*Rounding difference

The amounts presented above can be obtained directly by applying these


computations:

1. Assume that we would like to know the amount of the sinking fund accumulated
at the end of the third period, this could be obtained by computing the future
value of the sinking fund annuity payment. That is, sinking fund payment, SFP
becomes Pmt in the annuity formula. Hence Pmt = 1,329,197.83, n = 3 and r =
9%.
r
SFP = FV x (1 + r/f)n – 1

108
(1 + 0.09/1)3 – 1
FV = 1,329,197.83 x 0.09/1

FV = 4,357,243.41

2. On the fourth period, when we seek to know the amount of interest income, we
compute it by multiplying directly the amount of sinking fund accumulated by
the annual interest rate of 9%. That is:
Interest Income = 4,357,243.41 x 0.09
Interest Income = 392,151.91

3. The book value could then be easily computed by subtracting the original debt
the amount of sinking fund accumulated. That is:
Book Value = 10000000.00 - 4,357,243.41
Book Value = 5,642,756.59

In our previous example, the interest rate is assumed to have been constant for
the whole term of the investment. However, in practice, this is not always the
case. Hence, when interest rate varies during the investment’s term, the periodic
sinking fund deposits are adjusted to reflect the changes between the scheduled
and the actual interest income. Let us illustrate using an example.

Example 5: Using the same problem, assuming that the interest rate for the first
2 periods is 9%, then on the third and fourth period it becomes 10%
and eventually becomes 8% on the last two periods. Construct a
sinking fund schedule to reflect the adjustments.

Solution: Given that FV = 10,000,000, t = 6 years, f = 4 and r = 9%, sinking fund


payment is computed as:

r
SFP = FV x (1 + r/f)n – 1

0.09/1
SFP = 10,000,000 x (1 + 0.09/1)6 – 1

0.09
SFP = 10,000,000 x (1.09)6 – 1

SFP = 1,329,197.83

109
The adjusted sinking fund schedule appear as follows:

Period, end Expected Actual Difference Adjusted Periodic


Interest Interest Sinking Fund Increase in
Income Income Payment Fund
(1) (2) (3) (4) (5) (6)
1 0.00 0.00 - 1,329,197.83 1,329,197.83
2 119,627.80 119,627.80 - 1,329,197.83 1,448,825.63
3 250,022.11 277,802.35 27,780.24 1,301,417.60 1,579,219.94
4 392,151.91 435,724.34 43,572.43 1,285,625.40 1,721,349.74
5 547,073.38 486,287.45 -60,785.93 1,389,983.76 1,876,271.21
6 715,937.79 636,389.15 -79,548.64 1,408,746.49 2,045,135.65
Total 2024813 1955831.09 -68981.90 8,044,168.91* 10,000,.00000

*Rounding difference

The computations appear as follows:

1. Expected interest income appears as in the unadjusted schedule.


2. Actual interest income is obtained by multiplying the total amount of sinking
fund accumulated by the applicable interest rate. Thus, in period 1, interest is
zero because deposit is made at the end of the period. Period 2 actual interest is
obtained by multiplying 1,329,197.83 by 9%. On the third period, accumulated
sinking fund of 2,778,023.47 is multiplied by 10%, being the applicable rate to
arrive at 277,802.35 actual interest on the third period. The computation goes on
until the term of the investment using the applicable rate.
3. Periodic sinking fund deposits are adjusted to reflect differences between
scheduled/expected income with that of actual income. When the actual
income is greater, an investor would normally reduce deposits because of the
favorable accumulation of amount due to increased interest income. Otherwise,
the deposits would be more than the scheduled payment to reflect the lesser
amount of actual interest earned to meet the periodic increase of fund.

110
REVIEW QUESTIONS

1. What is a sinking fund?

2. What is amortization?

3. Give the reasons for sinking funds and amortizations.

4. How do you determine periodic payments in a sinking fund?

5. How do you compute amortization payments?

6. How does annuity relate with sinking fund and amortization?

111
PROBLEM 1. Amortization-Periodic Payments (Formula)

1. A loan of P20,000 is to be amortized by equal payments at the end of every


semi-annual for 5 years. If money is worth 5% compounded semi-annually,
find the periodic payment and show the schedule of payment using an
amortization schedule.

2. Tobey borrowed P40,000 with agreed interest of 9% annually paid in


advance. Determine the periodic payments and construct an amortization
table. Assume a term of 6 years.

3. Diego loaned an amount to be paid after 5 years. If the monthly payment is


P210.22 and interest is worth 5% monthly (every end of month), determine
the amount loaned.

4. Dora wishes to amortize an amount owed of P90,000. Determine the periodic


payment, and construct an amortization table if the loan was granted at
an interest rate of 4% compounded semi-annually for 5 years. Payment is
made every end of each 6 months.

5. Find the amount borrowed if a businessman periodically pays P4,210.16


at an interest rate of 4% compounded monthly. Assume payment is made
every month end for 3 years.

112
6. A loan of P50,000 is to be amortized for 2 years at the end of every quarter.
The interest rate is 4% compounded quarterly. Determine the periodic
payment and construct an amortization table.

7. A loan of P40,000 with interest at 4% compounded quarterly is to be


amortized by equal payments at the end of each 3 months for 1.5 years.
Find the periodic payment and construct the amortization table.

8. A debt of P10,000 with interest compounded semi-annually of 7% is


payable by 4 equal installments at the beginning of each six months. Find
the periodic payments and construct an amortization table.

9. Construct the amortization schedule for a P20,000 debt that is to be amortized


in 8 equal semi-annual payments at an annual rate of 12 % compounded
quarterly on the unpaid balance.

10. Consider a P100,000 debt that is to be amortized in 360 equal monthly


payments (i.e., 30-year mortgage) at a nominal rate of 6 % compounded
monthly on the unpaid balance. Construct an amortization schedule up to
6 months.

113
PROBLEM 2. Amortization-Periodic Payments (Formula)

1. A loan of P20,000 is to be amortized by equal payments at the end of every


semi-annual for 5 years. If money is worth 5% compounded semi-annually
and the agreed payment per period is P1,500, show an amortization
schedule.

2. Refer to Problem 1. Determine the amount of interest charged at the 3rd


amortization period. Determine the amount of outstanding principal at the
end of the 4th amortization period.

3. Boom-Boom borrowed P50,000 bearing an interest of 5% compounded


annually. The agreed payment is P7,000 annually except for the last payment
which would require a payment of P6,542.20. Construct an amortization
table to determine the time required of Boom-Boom to discharge the debt.

4. A loan is to be amortized by agreed equal payments of P4,000 at the end


of each month for 2 quarters except for the last or final payment which
would require the debtor to pay P5,654.90. If the interest is based on 10%
compounded monthly, determine the present value of the loan.

5. Mr. Ching bought a house worth P100,000 paying P40,000 down payment
and the balance to be paid in 6 equal monthly installment. Assume an
interest rate of 5% compounded monthly; determine the present value of
the installment payments. Compute for the outstanding principal at the end
of the 3rd payment. Find the remaining liability after the 4th payment.

114
6. A debt of P10,000 with interest compounded semi-annually of 7% is payable
by 4 equal installments at the beginning of each six months. If the agreed
payment is P2,000 each month, construct an amortization table. Determine
the size of the final payment.

7. A loan is being repaid with five semi-annual payments in advance. The


agreed payment is P5,000 each. If money is worth 5% compounded semi-
annually and the amount of loan is 25,000, construct an amortization
schedule.

8. Shane borrows P56,000, with interest at 9% compounded monthly. She will


amortize the debt by monthly payments of P2,000 each. Determine what
part of the 18th payment is the interest.
9. A debt will be settled by paying P5,600 at the end of each month for 3 years.
If the payments include interest at 6% payable monthly. Find the original
principal of the debt.

10. Refer to 8. Compute for the outstanding principal after the 3rd period.
Determine the part of the 12th payment as repayment of principal.

PROBLEM 3. Amortization-Comprehensive

Mr. Rob Aguinaldo received his monthly Statement of Account for an educational
plan bought from an insurance company for her daughter. He bought the plan 1
and a half year ago. He is reaching out your help because the printed copy seems
not so clear. He requested you to help him compute for the data regarding his
plan.

115
ORM 301.SOA

Insure’ Life
A Swiss Bank Subsidiary
Suite 125, 15/F Milestone Building
Makati Philippines

Account No. : RA9900-7788-9900


Planholder : Rob G. Aguinaldo
Mailing Address : 187, Calamba Road, Tagaytay City
Recipient : Kathrina A. Aguinaldo
Plan Availed : Insure’ Excel 99 (Educational)
Premium : P6,475.00
Payment Mode : PNB Account #1814-010406-240602

STATEMENT OF ACCOUNT

Period Outstanding Interest Amortization Principal Date Check Ref.#


Principal- due- Payment Repaid- Paid #
period end period period
end end
1 50,000.00 2500.00 6475.23 3975.23 06-30-10 01121 RA2254
2 46,024.77 1) 6475.23 2) 12-31-10 22378 RA8907
3 41,850.78 2092.54 6475.23 4382.69 06-30-11 65433 RA9988
4 3) 1873.40 4) 4601.83 DUE -- --
5 5) 6) 7) 8) -- -- --
6 9) 10) 11) 12) -- -- --
7 13) 14) 15) 16) -- -- --
8 17) 18) 19) 20) -- -- --
9 21) 22) 23) 24) -- -- --
10 25) 26) 27)* 28) -- -- --
Total 14,752.28 64,752.28

TOTYAL AMOUNT DUE: Principal P_____________29)


Interest P_____________30)
Total P_____________31)
*rounding difference
This is a system-generated data. Please disregard if payment has already been made.
Thank you.

VGarcia
VANESSA GARCIA
Billing Department

#665789000-0009-9988
Miles

Additional Questions:

a. What is the compounding frequency?

116
b. How long is the required amortization period expressed in years.

c. What is the amount due to be paid currently?

d. What is the periodic rate?

e. What is the annual rate?

f. What is the amount of plan applied for?

g. What part of the currently payable amount serves as principal repayment?

PROBLEM 4. Sinking Fund

Sinking Fund Payment Computation

Sinking Payment Time Periodic Interest Future


Fund Frequency Period Rate Compounded Value
Payment
Every six
2.5 years 5% Semi-annually P200,000
1. months
Every
8 quarters 4% Quarterly P400,000
2. quarter
Every
3 quarters 3% Monthly P230,000
3. month
Every 6 semi-
10% Annually P1,000,000
4. year annual
Every
2.75 years 5% Quarterly P15,000
5. quarter

117
Every
4 quarters 5% Monthly P40,000
6. month
Every six 3 semi-
12% Semi-annually P60,000
7. months annual
Every 12 quarters
13% Annually P50,000
8. year years
Every 3
5 quarters 6% Quarterly P30,000
9. months
2 quarters
Every
plus 2 2% Monthly P45,000
month
10. months

PROBLEM 5. Sinking Fund

Unless otherwise stated, periodic payments are made at the end of period.

1. Emily wants to invest an amount every month so that she will have P500,000
in 3 years to make a down payment on a new car. Her account pays 8%
compounded monthly. How much should she deposit each month? Construct
a sinking fund schedule.

2. In 6 years, you would like to have P900,000 for a down payment on a beach
house. How much should you deposit each quarter into a savings account
paying 3% interest compounded quarterly? Construct a sinking fund schedule.

118
3. The Philippine government has P5,000,000 worth of bonds that are due in 20
years. A sinking fund is established to pay off the debt. If the state can earn
10% annual interest compounded annually, what is the annual sinking fund
deposit needed? Construct a sinking fund schedule.

4. The PAGCOR is required by law to set aside funds to replace its building.
It is estimated that the new building will need to be replaced in 20 years at
a cost of $3,900,000. The PAGCOR can invest in treasuries yielding 6% paid
semiannually. If the PAGCOR invests in the treasuries, what semiannual
payment is required to have the funds to replace its building in 20 years?

5. A P60,000 debt is to be repaid after one year. The debtor wishes to establish a
sinking fund that yields 7% interest compounded monthly. Construct a sinking
fund schedule.

PROBLEM 6. Sinking Fund

1. In January 1, 2012, the Japanese government lent the Philippine government’s


Finance Department funds enough to construct a flood control project for the
provinces of Pampanga and Bulacan upon submission of a comprehensive
proposal and upon compliance with certain conditions attached to the statutory
agreement. The Philippine government issued 900, $1,000 worth of bonds to
be paid on 2022. The debt agreement provides for a cost of 5% compounded
quarterly. What quarterly payment is required of the Philippine government
to accumulate the funds needed to retire the bonds in 2022? Assume 1$=P45

2. Refer to Problem 5. Without constructing a sinking fund schedule, what is peso


value of the accumulated balance of the fund on the year 2014?

119
3. Refer to Problem 5. What is the yield (in peso) in the fund for the year 2015?
2016?

4. Refer to Problem 5. After the payment of the bonds, how much of the sinking
fund will earn interest?

5. Magnolia Corporation floated certain amount of bonds. As an entity policy, a


sinking fund retirement fund was established requiring P186,081.13 quarterly
payments for 5 years. If the fund pays 12%compounded quarterly, how much
bonds were floated?

PROBLEM 7. Sinking Fund

1. 2000 Miles, Inc. wants a sum of P2,000,000 after 3 years to retire a debt. If the
company will invest in a fund that pays 5% compounded quarterly, what is the
amount of quarterly payments? Construct a sinking fund schedule.

2. Breakwater Corporation owed Banko Filipino an amount with maturity value


of P900,000. If the Board of Directors decided to establish a sinking fund for
that purpose, what amount of monthly payment should be made assuming a
fund pays 10% compounded monthly and the loan is to be paid 5 years from
now?

3. Refer to Problem 2. Construct a sinking fund schedule.

120
4. The management of Aurora Bank, one of the most stable banks in the industry
recently acquired Echague Bank due to the financial difficulties faced by the
latter. As a part of the takeover, the management has decided to relocate the
office of Echague Bank with a new name, Salay Bank. The management decided
to establish a sinking fund for the construction of a new building 5 years from
now. The construction will require an outflow of P500,000,000. If they invest in
a fund that pays 15% monthly, what is the required periodic fund payment?

5. Refer to problem 4. Construct a sinking fund schedule showing the first 6


periods.

PROBLEM 8. Sinking Fund-Comprehensive

The management of SVV & Co. received the following statement from a trustee
for a sinking fund established to retire a certain debt. This statement shows a
completed investment, enough to settle the currently maturing debt. Assume that
we are at period 0, compute for the missing data.

Complete the table below and answer the following questions:

Hint: Fix your calculator into 2 decimal places. In a year, the account is funded 4 times evenly.

Interest in Periodic Accumulated


Payment Sinking Fund Book Value
Sinking Increase in Amount of
Period, n Payment of the Loan
Fund Fund Sinking Fund
0 0 0 0 0 1.
1 2. 0 1,409,241.71 1,409,241.71 3.
2 4. 42,277.25 5. 6. 7.
3 1,409,241.71 85,822.82 1,495,064.53 8. 15,644,175
4 1,409,241.71 9. 1,539,916.47 10. 14,104,258
5 1,409,241.71 11. 12. 13. 14.
6 1,409,241.71 224455.6687 1,633,697.38 9,115,553.00 10,884,447

121
7 1,409,241.71 15. 1,682,708.30 16. 17.
8 1,409,241.71 323947.8391 18. 19. 20.
9 1,409,241.71 375943.5255 21. 14,316,636.09 22.
10 1,409,241.71 23. 24. 25. 3,844,623
11 1,409,241.71 484661.3063 1,893,903.02 26. 1,950,720
12 1,409,241.71 541,478.40 27. 28. 29.
Total 16,910,900.51 3,089,099.49 30.    

Additional Questions:

a. What is the periodic rate?

b. What is the effective annual rate?

c. What is the compounding frequency?

d. What is the maturity value of the loan?

e. Why does the payment in period 1 earned no interest?

f. What type of annuity is seen in the table?

122
References
Agcaoili, Z. A. et. al. (2001). Business Operations: Mathematics of Investment.
Valenzuela City: Mutya Publication
Ballada, W. L. (2009). Investment Mathematics Made Easy. Manila: DomDane
Publishers
Caras, M. S. et. al. (2008).Mathematics of Investment Revised Edition. Manila:
Booklore Publication
Florendo, D. R. R. (2006). Mathematics of Investment: Math 240 2nd Edition. CLSU,
Nueva Ecija
Gabriel, P. B. et. al. (1994). Fundamentals of Investment Mathematics Revised Edition.
Manila: Island Publishing House
Hernandez, R. M. et al. (1995). Mathematics of Investment Workbook with Tables.
Pasig City: Academic Publishing
Liquigan, R.M. (2001). Mathematics of Investment: Worktext. Cabanatuan City:
Love Printing and Publication House
Narag, E.R. (2006) Mathematics of Investment. Makati City: F & J De Jesus
Naval, V. C. et. al. (2010). Mathematics of Investment 2nd Edition. Quezon City
Nocon, F. P. et. al. (2000). Modern Mathematics: Calculator-based with
Applications. Mandaluyong City: National Bookstore

123
124
TABLES

125

You might also like